Mcq Ophthalmology

December 8, 2016 | Author: Mohammad M.Omran | Category: N/A
Share Embed Donate


Short Description

Download Mcq Ophthalmology...

Description

1) i i. LAMKIN LAMKIN A. . NEURO-OPHTHALMOLOGY NEURO-OPHTHALMOLOGY L1 . Photostress recovery test Photostress recovery test a. This simple clinical test may help to differentiate central visual loss cause d by a macular lesion from that derived from optic neuropathy. Each eye is tested separately. b. Best corrected visual acuity is measured (the test is accurate only with acui ty of 20/80 or better), after which the patient is instructed to gaze directly into a strong light (eg, direct ophth almoscope or slit-lamp beam) held 2-3 cm from the affected eye. c. The patient is then directed to read the previously measured best acuity line as soon as possible. d. Normal recovery time is typically 45-60 seconds, and this is maintained with optic nerve disease. e. In patients with maculopathy, however, recovery times are frequently 90-180 s econds or more. L2 A pattern visual evoked response (VER) a. The two crucial parameters used for functional evaluation include the height of the first positive or upward wave (amplitude) and the time between stimulus presentation and the appearance o f this wave (latency). Implicit time, which is the time it takes to reach peak amplitude after a stimulu s is applied. b. (As opposed to a flash VER) is required for visual acuity assessment in preve rbal children. c. Although the VER is useful in establishing factitious visual loss, its reliab ility is limited by the fact that patients can produce false readings by using accommodation to fog their vision. d. Abnormalities in VER latency and amplitude have been reported in various macu lopathies and retinopathies, and, therefore, these features cannot distinguish optic neuropath y from retinal disorders with complete reliability. L3 Although OKN, the rocking mirror test and base up prism can help discover fac titious monocular visual loss, these tests are not sensitive enough to diagnose factitious visual deficit at 20/100 l evel. For mild visual deficit, a fogging refraction, stereo acuity, and red-green glasses may be useful in diagnosing fac titious visual loss. L4 An optic tract lesion may cause unilateral decrease VA if the lesion is not c omplete or if the optic nerve or chiasm is also involved. If the optic nerve or chiasm is involved, the APD is typically ip silateral. Pure optic tract lesion can produce a contralateral APD as well as a complete homonymous hemianopia. L5 In the setting of upgaze paresis, upturning of the eyes on forceful opening o f closed eyelid (intact Bell's phenomenon) implies supranuclear lesion. Upturning of the eyes upon forceful ope ning of closed eyelids is known as the Bell phenomenon. If forcefully opening closed eyelids stimulates upgaze, the final common pathway (infranuclear) for upgaze must be intact. L6 Some tissue such as cortical bone, rapidly flowing fluid (blood) and air give s no signal at all on MRI {dark (hypointense)}. T1-weighted images tend to show the anatomy well, whereas T2-wei ghted images tend to show the

pathology well. L7 The macular fibers constitute a large portion of the chiasm and decussate in the posterior chiasm. The chiasm lies approximately 1 cm (not 1 mm) above the anterior pituitary gland. L8 Optokinetic nystagmus (OKN) abnormalities indicate lesions of the parietoocci pital (slow-phase pursuit abnormalities) or the frontal lobe (fast-phase recovery abnormalities). High con gruity of visual field deficits indicates a lesion in the occipital lobe. Both partial complex seizures and formed visual hallucinations may be seen with temporal lobe lesions. Inferior nerve fibers from the superior retina course ant eriorly in Meyer's loop; therefore, lesions affecting Meyer's loop will result in pie-in-the-sky defects contralater al to the lesion. L9 Although OKN abnormalities asymmetry may occur rarely with occipital lobe les ion, this finding is generally indicative of parietal lobe lesion. L10 Causative events in the pathophysiology of optic disc oedema include: swolle n axons, intracellular fluid accumulation, and interruption of axonal transport. L11 The most typical visual field finding in acute (not chronic) papilloedema is an enlarged blind spot. Although rare unilateral papilloedema may occur, for example, if contralateral optic atrophy e xists, papilloedema may be detectable only in a viable disc. L12 Pseudotumor cerebri is characterized by (i) increased intracranial pressure on lumbar puncture, (ii) normal neuroimaging studies (although the ventricles may be small), and (iii) normal ce rebrospinal fluid (CSF). Papilledema need not be present for the diagnosis. Although the neurologic exami nation is usually normal, sixth nerve palsy may occur with increased intracranial pressure of any etiology. L13 Indications of treatment of pseudotumour cerebri are severe headache and vis ual field loss. Obesity is not an indication for treatment, although weight loss (even as little as 6% of total bo dy weight) often improve the condition. L14 The most common cause of permanent visual loss in patients with cavernous si nus-dural fistulae is open-angle glaucoma. Studies have documented that up to 80% of patients with cavernous sinu s-dural fistulae will develop ocular hypertension. Twenty-five percent of patients will develop optic disc cup ping and 20% visual field defects. Any entity that raises episcleral venous pressure can cause secondary open-angle glaucoma. L15 GCA: a. 40% of untreated patients will develop some form of permanent visual loss.

b. 65% of untreated patients will have contralateral involvement after permanent visual loss in one eye. c. The second eye may still become involved in 25% of cases despite early and ad equate steroid administration, usually within 6 days of starting treatment. Visual loss is usua lly profound and is unlikely to improve even with immediate treatment. d. 20% of patients do not have systemic symptoms (i.e. occult GCA). L16 Nonarteritic anterior ischemic optic neuropathy (NAION) is far more common t han arteritic anterior ischemic optic neuropathy (AAION) (approximately 95% vs. 5%) and patients have a lower mean age at diagnosis than patients with AAION (60 years vs. 70 years). It usually occurs in a younger age group and may resemble optic neuritis. L17 The various ways to differentiate the NAION from optic neuritis include (i) the absence of pain with eye movement, (ii) the age group affected, and (iii) delayed optic disc filling present in 75% of NAION cases (whereas filling should be normal in optic neuritis). L18 Although aspirin effective in reducing systemic vascular events and is frequ ently prescribed in patients with NAION, it doesn t appear to reduce the risk of involvement of the fellow eye. L19 Nearly 100% of patients dying of multiple sclerosis had some degree optic ne rve demyelination. L20 Ninety percent of optic gliomas occur in the first two decades of life. The percentage of patients with optic nerve glioma that have associated neurofibromatosis (NF-1) ranges from 14% to 60%. Mal ignant gliomas of the visual pathways, although rare, occur more frequently in middle-aged adults than in chi ldren. Survival averages 6 to 12 months after diagnosis. L21 Yanoff and Kanski. Gliomas of the optic nerve develop in 10 15% of affected pa tients. They can occur unilaterally or bilaterally and frequently involve the optic chiasm. L22 Optociliary vessels occur in optic nerve meningioma, CRVO, sphenoid wing men ingioma, long standing POAG, optic glioma and chronic papilloedema. L23 In contrast to optic nerve gliomas, meningiomas occur primarily in adults an d are three times more common in women. Although persons with neurofibromatosis (NF-1) have a higher incidence of meningiomas than the general population, only a few patients with meningiomas have NF-1. With contrast comput ed tomography (CT) scanning, the peripheral part of the involved optic nerve may show enhancement, resulting in the railroad track sign ( kinking is specific for optic nerve glioma). L24 Dominant (Kjer) optic neuropathy (DOA) manifests between 5 and 10 years of a ge. Visual loss may progress until the mid-teens, at which point it usually stabilizes. Color defects are alm ost universally present, and tritanopia (which can be detected with the Farnsworth-100 hue testing) is suggestive of DOA . Inheritance is naturally autosomal dominant. L25 Leber's hereditary optic neuropathy (LHON). a. All offspring of a female carrier are either affected or carriers. b. Ten percent of female carriers will be affected. c. There is generally sequential asymmetric bilateral involvement. d. A small percentage of patients will enjoy partial or complete recovery late i n their course. The incidence of

spontaneous recovery has been reported to be as high as 10%. L26 Patients with papillophlebitis have normal or near normal visual acuity. It ma y be a form of incomplete central retinal vein occlusion (CRVO) and usually resolves spontaneously within 12 month s. L27 Optic disc drusen occurs almost exclusively in white individuals. They are g enerally bilateral. L28 Morning glory anomaly F: M is 2:1. L29 Non-secreting tumors often present with visual field loss, whereas secreting tumors present with endocrine dysfunction. An exception is prolactin-secreting tumor in male patients because the decreased libido and impotence are often not reported early in their course. L30 When the causative lesion is not located at the site responsible for a clini cal sign, it is described as false localizing. Bitemporal macular hemianopia can only arise from compression of the posterior chiasm. Although most lesions responsible for this compression are parasellar, a lesion distant f rom this site (e.g., a tumor at the base of the brainstem) also may lead to chiasmal compression. This occurs if the lesion causes obstructive hydrocephalus, which enlarges the third ventricle, compressing the chiasm. L31 Precipitous aneurysmal distention caused by hemorrhage of the posterior comm unicating artery at its junction with the internal carotid artery may occur, creating a third nerve palsy. Ninety -five percent include pupillary involvement, and pain is nearly always present, although not universally. Furthe rmore, many third nerve palsies resulting from diabetic vasculopathy also can be intensely painful. L32 Third nerve aberrant regeneration never occurs with diabetic oculomotor neur opathy. Aberrant regeneration of the third nerve implies another etiology, such as aneurysm, tumor, inflammation, or trauma. L33 The most common cause of acquired fourth nerve palsy in adult is trauma. L34 The three-step test is useful for diagnosis but does not differentiate betwe en congenital and acquired trochlear nerve palsy. Large vertical fusional amplitudes (>5 prism diopters) (greater tha n 3.. AAO) and facial asymmetry from childhood head tilting suggest a decompensated congenital lesion. L35 Congenital fourth nerve palsies may be revealed by old photographs that show head tilting or a large vertical fusional range.

L36 35-year-old Chinese man presents with right ptosis and an abduction deficit, facial hypesthesia, and keratoconjunctivitis sicca in the right eye. This picture is highly suggestive o f nasopharyngeal carcinoma. L37 Nasopharyngeal carcinoma can involve numerous cranial nerves because of its proximity to the prepontine basal cistern. Most frequently, the trigeminal nerve is involved, causing facial hypes thesia or facial pain. The abducens nerve is the second most common. The hallmark of nasopharyngeal carcinoma is its propensity to involve multiple cranial nerves non-contiguously. Nasopharyngeal carcinoma is common in Chinese men. The least differentiated forms are also known as Schmincke's and Regaud's tumors. L38 Horizontal saccades originate in the contralateral frontal lobe, but either hemisphere can produce ipsilateral saccades if the other hemisphere is damaged. Appropriate stimulation of the pari etal or occipital cortex can also produce contralateral saccades. L39 Oculomotor apraxia. a. Both congenital and acquired forms of oculomotor apraxia may be observed. b. Saccades are generally affected more than Pursuits. c. Horizontal movements are generally affected much more than vertical movements . d. In the acquired form, blinks are frequently used to break fixation. e. In the congenital form, children frequently use compensatory, exaggerated hea d turns to refixate. L40 Opsoclonus and ocular flutter, ocular motor disorders most commonly associat ed with malignancy (neuroblastoma in children and small cell lung carcinoma in adults), these movem ents may be the first presenting sign of malignancy. L41 MS may cause ocular flutter. L42 Ocular bobbing is most commonly seen in comatose or quadriplegic patient wit h large infarcts or brain stem haemorrhage. L43 In progressive supranuclear palsy downward are the first affected (saccades are affected than pursuit); in contrast to Parinaud dorsal midbrain lesion upward is firstly affected. L44 Latency for smooth pursuit is shorter (125-135 ms), than saccades which requ ire (150-200 ms). L45 Parinaud syndrome. a. Collier syndrome may worsen with attempted upgaze. b. Skew deviation and papilloedema may be seen depending on the etiology. c. Convergence-retraction nystagmus is also a response to an effort at upgaze th at triggers medial rectus contraction. Hence this form of nystagmus is worsened by upward OKN testing. L46 Monocular nystagmus in toddlers has been associated with chiasmal glioma and hypothalamic glioma. It is also seen in blind eyes, MS, and spasmus nutans. L47 According to Alexander's law, the nystagmus is more pronounced when gaze is directed toward the side of the fast-beating component. L48 Of various forms of nystagmus, downbeat nystagmus most frequently violates A lexander's law. L49 Downbeat nystagmus is almost always present in primary position, but in acco rdance with Alexander's law, the down beating movements are accentuated in downgaze (especially down gaze to eith er side). Patients usually report difficulty in reading and oscillopsia. The oscillopsia can be debilitatin

g. (AAO) L50 The nystagmus most likely to be localized is see-saw nystagmus. It results f rom third ventricle tumours or diencephalic lesion involving the connections to the interstitial nucleus of Caj al and is therefore most localized. L51 See-saw nystagmus is most frequently associated with bitemporal hemianopia, because posterior chiasm is most vulnerable to diencephalic tumour. L52 Vergence eye movements are felt to have a pathway of their own for stimulati ng the brain stem motor nuclei. L53 Upper motor neuron facial nerve paralysis usually leaves voluntary eyelid cl osure intact. Voluntary eyelid closure is preserved in upper motor facial nerve paralysis, because of bilateral supranucle ar input to the upper fascial muscles (orbicularis). L54 Spontaneous or reflexive blinking requires normal function of the basal gang lia, as do other nonvolitional facial expressions. Therefore, these are commonly affected in Parkinson's disease. Voli tional movements, in distinction, are generally not adversely influenced. L55 Marcus jaw winking reflex is an example of synkinesis (abnormal innervation connecting two groups of normally unrelated muscles, such that contract together). L56 Compression of the facial nerve at the cerebellopontine angle has been demon strated in 90% of cases of hemifacial spasm. L57 Approximately 85% of patients with Bell's palsy spontaneously recover, altho ugh some will have some degree of aberrant regeneration. Recovery usually begins within 3 weeks of diagnosis and u sually becomes complete by 2 to 3 months. L58 The facial nerve is the most frequently involved cranial nerve in neurosarco idosis. The parotid gland can become infiltrated with granulomas, and the facial nerve is involved at this site. Faci al nerve involvement in sarcoidosis is frequently bilateral but asymmetric. The second most commonly involved crania l nerve is the optic nerve. L59 Of the three most common causes of facial nerve overactivity, only essential blepharospasm is believed to be related to basal ganglia dysfunction. Compression of the facial nerve in the cer ebellopontine angle by anomalous vessels has been demonstrated in 90% of cases of hemifacial spasm. Tumors in the cerebellopontine angle also

can cause hemifacial spasm. Facial myokymia is caused by disease in the pons inv olving the facial nucleus or fascicle. The most common causes include multiple sclerosis (MS) in adults and p ontine glioma in children. L60 The following conditions may be confused with essential blepharospasm. a. Severe dry eye. b. Retained conjunctival foreign body. c. Tardive dyskinesia. L61 Hemifacial spasm is distinct from essential blepharospasm in that it is unil ateral, and multiple facial muscles are involved. L62 Pupillary dilatation may be the only sign of third nerve palsy in uncal hern iation and basilar meningitis. Total oculomotor nerve palsy sparing the pupil in elderly patients usually suggests mi crovascular aetiology. L63 DDX of tonic pupil includes herpes zoster, syphilis, and giant cell arteriti s. L64 Localization of the lesion in Horner's syndrome is part of the clinical work up and sometimes guided by the extent of anhidrosis. First-order lesions (central nervous system) cause ipsilateral anhid rosis of the entire body. Secondorder lesions (Pancoast's tumor, neck trauma) cause ipsilateral facial anhidrosi s. Third-order lesions cause anhidrosis only around the affected eye or none at all. L65 Painful Horner syndrome may be caused by many disorders (neck trauma, migrai ne, cluster headache), but spontaneous dissection of the common carotid artery must be ruled out with angio graphy or MRI/MRA. L66 After determination of which pupil is abnormal, the next step in evaluating a patient with anisocoria is slit-lamp examination. Most patients with nonphysiologic anisocoria have sphincter muscle dysfunction visible at the slit lamp. Trauma is most common, followed by Adie's pupil. L67 The risk of developing multiple sclerosis (MS) for a relative of a patient w ith MS is nearly 20 folds higher than the general population. MS is a multifactorial disease with both genetic and environ mental risk factors. L68 The most common cranial mononeuropathy in MS is optic neuritis. The most fre quently affected motor nerve is the sixth nerve. L69 5-10% of patients with MS will have findings of posterior uveitis, including pars planitis or retinal periphlebitis. L70 Pheochromocytomas produce, secrete, and store catecholamines. They are most often derived from the adrenal medulla but may arise in any of the sympathetic ganglia. Two phakomatoses, neuro fibromatosis, and von HippelLindau disease, are associated with the tumor. L71 Astrocytic hamartomas can be seen in tuberous sclerosis or neurofibromatosis , or they can be sporadic. They are located in the nerve fiber layer of the retina and sometimes can be associated w ith the optic disc. L72 The triad of adenoma sebaceum, mental retardation, and seizures is considere d pathognomonic for tuberous sclerosis. This is known as Vogt's triad and is present in 30% of patients with tuberous sclerosis. Facial angiofibromas (adenoma sebaceum) are present in at least 75% of adults with tube rous sclerosis. L73 Seizures are part of the classic triad in tuberous sclerosis (80% of patient s have seizures). Patients with Sturge-

Weber syndrome and meningeal hemangioma may have seizure disorders. Patients wit h neurofibromatosis also have an increased risk of seizures. L74 Unilateral congenital glaucoma is seen in 25% of cases of Sturge Weber syndr ome. NF1 associated juvenile glaucoma. Glaucoma may be associated with V. H. L. disease. L75 Heart block in patient with Kerns Sayre syndrome is lethal and among the las t signs to develop. Some studies indicate that elevated cerebrospinal fluid (CSF) protein levels correlates with the presence of heart block. L76 A diagnostic feature of myotonic dystrophy is the presence of polychromatic ( Christmas tree ) cataracts, which may present even in the amyotonic forms. Wilson's disease is associated with the s unflower cataract. L77 Lid retraction and lag is the most common sign of Graves's disease. L78 The most frequently involved muscle in dysthyroid orbitopathy is the inferio r rectus. The medial rectus is the second most frequently affected muscle and may simulate a sixth nerve palsy. The least frequently involved muscle in Graves' ophthalmopathy is the lateral rectus. L79 Myasthenia gravis. a. 75% of MG patients will have eye findings at presentation. b. Only 33-50% will have ocular myasthenia only. c. 90% will have ocular symptoms during the course of the disease. d. Ptosis is the most common. e. 5% will develop Graves' disease. f. 10% will have thymus enlargement on CT scan. g. Ocular myasthenia should remain localized for 2 years to be reassured that sy stemic disease is unlikely to develop. L80 A tiny fraction of patients with Graves' disease will develop MG. L81 IOP after high grade carotid stenosis are low (caused by ciliary hypoperfusi on), normal, or elevated (caused by neovascularization of the iris and angle). Unilateral arcus is highly suggestive of contralateral carotid disease. Carotid stenosis protects the ipsilateral cornea from serum lipid deposition. L82 The etiology of Coats' disease is unknown, and there does not appear to be a ny genetic, familial, racial, or ethnic predisposition. However, Coats'-type retinal vascular changes have been noted in patients with facioscapulohumeral muscular dystrophy, Turner's syndrome, Senior-Loken syndrome , and one variant of the

epidermal nevus syndrome. In addition, Coats'-like retinopathy has been noted in up to 3.6% of patients with retinitis pigmentosa. L83 Downbeat nystagmus in primary position is localized to the craniocervical ju nction (or certain intoxications). This patient's clinical symptoms of intermittent occipital headaches with sudden head movements or anger suggest the diagnosis of Arnold-Chiari malformation. Arnold-Chiari malformation is one of th e most common causes of downbeat nystagmus. L84 Clonazepam and suboccipital craniotomy have been used in the treatment of do wnbeat nystagmus in patients with Arnold-Chiari malformation. By contrast, carbamazepine toxicity has been associa ted with downbeat nystagmus. L85 A 9-year-old boy presents to the ophthalmologist with the complaint that he has lost his position on the school basketball team because he cannot see the basket. He notes occasional morning he adaches but denies any nausea or vomiting. Examination reveals visual acuity of 20/40 in the right eye and 20/25 in the left eye. The patient has marked symmetric weakness of upgaze bilaterally. His pupils are 7 mm and are poorly reactive to light, with better reaction to a near target. There is approximately 2 mm of superior sclera l show bilaterally. Fundus examination suggests optic atrophy in both eyes. A review of the systems reports increased consumption of water, with frequent urination at night. The most likely diagnosis is: pinealoma. a. This patient appears to have Parinaud's dorsal midbrain syndrome, which may i nclude the following findings: pupillary light-near dissociation, lid retraction (the Collier sign), upgaze paresis, convergenceretraction nystagmus, fixation instability, small-amplitude skew deviation, and papilledema (if ventricular outflow has been compromised). The most common cause in this age group would be a pinealoma. Other causes include stroke, hydrocephalus, and multiple sclerosis (MS). b. After neuroimaging has been obtained, a lumbar puncture would be an important step in the diagnostic evaluation of this patient because pinealoma classically sheds cells into the ce rebrospinal fluid (CSF). L86 Optic neuritis in childhood is more commonly bilateral. Visual loss can be s evere, although intravenous corticosteroids can improve visual function. Diffuse enlargement of the optic ne rve on computed tomography (CT) scan may be seen in this condition, mimicking a neoplasm of the optic nerve sheath. The demyelination typically follows a viral illness or vaccination by 10 to 14 days. L87 The differential diagnosis for posterior ischemic optic neuropathy (PION) sh ould include radiation optic neuropathy, status post-coronary artery bypass graft, anemia, acute systemic hypotension, gi ant cell arteritis (GCA), and syphilis. Well-controlled essential hypertension is associated with anterior isc hemic optic neuropathy (AION) but not PION. L88 Bilateral disc oedema and headache may be caused by several reasons, but hyp ertension should be the first to be excluded because it is quite easy to do so. After checking blood pressure, a neu roimaging should be obtained

immediately. L89 Meningothelial (syncytial) meningioma is the most common histopathologic typ e of meningioma seen within the orbit. Pilocytic describes the cell type of gliomas of the visual pathways. L90 A meningioma arising in optic nerve sheath is most likely to lead to optic d isc edema. L91 In general, optic nerve compression >1 cm posterior to the globe does not ca use disc edema. L92 Tobacco-alcohol amblyopia seems to be seen only in heavy smokers/drinkers wi th poor nutrition. This has led many to believe that a combination of toxic plus nutritional insults must be nec essary for the development of the disorder. L93 The most common etiologies of bilateral central or cecocentral scotomas incl ude hereditary optic neuropathy and nutritional optic neuropathy (vitamin B12 and folate deficiency), drug toxicity, tobacco-alcohol amblyopia, and infiltrative disorders such as syphilis and tuberculosis. Cyanide levels are not helpful in suspected tobacco-alcohol amblyopia. L94 Although disc edema, disc hemorrhages, and choroidal rupture may be seen in acute traumatic optic neuropathy, the most common finding is a normal fundus. Disc pallor would be unusual in the acute setting but present in all cases after several weeks. L95 The development of diabetic papillopathy appears to be independent of serum glucose levels. Diabetic papillopathy is classically seen in young adults with type 1 diabetes with moderate to severe retinopathy (although it can be seen in patients with type 2 diabetes as well). It is painless, and associated v isual loss is generally mild. The disorder generally resolves spontaneously. L96 Thyroid optic neuropathy is considered to be a compressive optic neuropathy because of enlargement of extraocular muscles at the orbital apex. Treatment of thyroid optic neuropathy m ay include orbital radiation (usually 1,500 to 2,500 rad over a 10-day period) and orbital decompression, which provid es the most potential for decompression of the optic nerve. Systemic corticosteroids are thought to be eff ective only in the acute congestive phase and not in the fibrotic period. Therefore, if no response is noted within 3 weeks, systemic steroids should be tapered and another modality should be considered. Although a subtotal thyroidec tomy may provide primary treatment of dysthyroid state, it will have no effect on the eye findings (excep t, perhaps, lid retraction). L97 Bromocriptine has been shown to be effective primarily in the management of prolactin-secreting pituitary tumors and is less effective or ineffective with other types of pituitary tumors. L98 Acute visual loss associated with severe headache is a typically a sign of p ituitary apoplexy.

L99 Fortification spectra that precede visual loss suggest a diagnosis of migrai ne headaches. A fixed visual defect suggests a cerebral arteriovenous malformation (AVM). A family history of headac hes and nausea associated with visual disturbances are more suggestive of a migraine. Field defects may be foun d during acute migraine headaches, but those present interictally suggest another diagnosis. L100 A skew deviation is a motility disturbance with a vertical component that d oes not have a pattern consistent with a discrete muscle underaction or nerve palsy. They are generally due to supranucle ar or vestibulo-ocular dysfunction and generally reflect brainstem disease. They are typically comitant but not alw ays. L101 Small vessel disease is the most common cause of unilateral sixth nerve pal sy. More common causes of bilateral sixth nerve palsy include increased intracranial pressure, head trauma, and tumo rs of the ventral brainstem. Congenital bilateral sixth and seventh nerve palsies are characteristic of M.biu s' syndrome. L102 Isolated sixth nerve palsy in children is most commonly attributable to pos tviral inflammation occurring 1 to 3 weeks following a nonspecific viral illness of the upper respiratory tract. Recovery i s generally complete and occurs within 10 to 12 weeks. L103 Medial rectus contracture is distinctly uncommon in Duane's syndrome. In co ngenital sixth nerve palsy, it is quite common and results in esotropia in primary position. To differentiate between Du ane syndrome and congenital sixth nerve palsy is orthotropia in primary position because medial rectus contracture is uncommon in Duane syndrome, while sixth nerve palsy result in esotropia in primary position. L104 Disruption of the medial longitudinal fasciculus (MLF), which carries proje ctions of interneurons from the contralateral sixth nerve nucleus to the ipsilateral medial rectus subnucleus, r esults in ipsilateral absence or slowing of adduction and contralateral abduction nystagmus. This combination of findings is termed internuclear ophthalmoplegia (INO). Vertical nystagmus and skew deviations are frequently fou nd in association with INO but are not universal. L105 Bilateral internuclear ophthalmoplegia (INO) is more frequent in demyelinat ing disease than in cerebrovascular disease. This is because the brainstem blood supply is lateralized right and left circulations are usually discrete and end at the midline. Demyelinization does not respect the midline. L106 Exotropia in primary position can occasionally occur in association with a bilateral internuclear ophthalmoplegia (INO), resulting in a syndrome called wall-eyed bilateral INO. L107 Miller-Fisher syndrome is generally considered a variant of Guillain-Barré sy ndrome that results in ophthalmoplegia, ataxia, and areflexia. Serum immunoglobulin G (IgG) autoantibod ies and elevated cerebrospinal fluid (CSF) protein may be present. Complete recovery is common. L108 The presence of optic nerve dysfunction, manifested by decreased vision, an afferent pupillary defect, and/or dyschromatopsia, distinguishes an orbital apex syndrome from a cavernous sinus s yndrome because the optic nerve passes through the optic canal and does not enter the cavernous sinus. L109 Tolosa-Hunt syndrome.

a. Idiopathic, sterile inflammation that primarily affects the cavernous sinus. b. Pain is almost always present. c. Painful ophthalmoplegia. d. Sensory deficit of the trigeminal nerve (most commonly ophthalmic branch). e. Neuroimaging may show a mass, which often is enhanced with contrast. f. Patients with Tolosa-Hunt syndrome respond dramatically to corticosteroids. g. Normal MRI is unlikely but not impossible. h. It is DX of exclusion. L110 Iris and posterior segment neovascularization, as well as rapidly progressi ve cataract, may all be seen as complications of the ischemic oculopathy that these fistulae generate. Corneal e xposure caused by proptosis is another potential complication of carotid-cavernous fistula. L111 Both high-flow and low-flow carotid-cavernous fistulae may be associated wi th a history of head trauma. Low flow carotid cavernous fistula may also be associated with minor head trauma. L112 59-year-old man presents to the emergency room with a complaint of sudden-o nset oscillopsia and diplopia. Examination reveals an alcohol odor to his breath, normal acuity, bilateral abdu ction deficits, and coarse binocular nystagmus. Appropriate intervention should include intravenous thiamine. Acute t hiamine deficiency (Wernicke's encephalopathy) can result in central scotomas as well as ophthalmoplegia, prima rily affecting cranial nerves III and VI. It can be precipitated in nutritionally depleted alcoholics given intravenou s glucose alone because of sudden consumption of systemic thiamine stores. L113 Giant cell arteritis (GCA) is common in northern European climates. On the basis of autopsy studies, the prevalence has been estimated to be 1.1% of the Scandinavian population. L114 Clinical characteristic of chiasmal compression includes: a. Field abnormalities most notably centrally with fainter test object. b. Postfixation blindness. c. Temporal color desaturation. d. Hemifield slip result in diplopia. L115 The examiner attempts to confirm diagnostic suspicions by eliciting the Pul frich phenomenon. To do this, the examiner: asks the patient to watch the pendulum on the grandfather clock across the room and report any three-

dimensional movement. The Pulfrich phenomenon probably reflects delayed conducti on in the demyelinated nerve. Oscillating objects perceived by the affected eye appear to be behind the image seen with the healthy eye, simulating three-dimensional movement where there is only movement within one pl ane. The delayed implicit time is the electrophysiologic correlate of the bizarre perception known as the Pulfr ich phenomenon. L116 LHON, telangiectatic peripapillary vessels not leak on FA. L117 Sumatriptan is a serotonin antagonist available orally, by injection, or as a nasal spray. The -triptan antimigraine drugs are used for symptomatic relief of migraines but are contraindicated in pa tients with basilar artery migraine. They can produce myocardial infarction (MI) and should be used with caution in p atients with severe hypertension or coronary artery disease. L118 The best initial choice for prophylaxis of acute, severe migraine headache is propranolol. Methysergide is an ergot alkaloid with numerous potential side effects. The beta-blockers are effective a nd safer. L119 The Ischemic Optic Neuropathy Decompression Trial of IONDT for NAION reveal ed no significant benefit for treatment (improvement in visual acuity by at least three Snellen lines: 32.6% f or treated patients vs. 42.7% for controls}. Moreover, the treatment group showed a statistically significantly gr eater risks of acuity that worsened by three Snellen lines or more (23.9% treated vs. 12.4% control). IONDT is not curr ently recommended for the treatment of NAION. It has been suggested that levodopa has a beneficial effect, but this remains unconfirmed. L120 Pseudotumor cerebri typically (not universally) presents with visual loss a nd headaches. Neurologic abnormalities are usually absent except for occasional abducens paralysis. Although the openin g pressure is elevated, cerebrospinal fluid (CSF) protein levels are either normal or low. Many cases ar e probably based on decreased reabsorption of CSF. L121 Binding antibodies to acetylcholine receptors are found in 90% of patients with generalized myasthenia gravis (MG). L122 Relative to generalized myasthenia, ocular myasthenia is completely localiz ed to ocular or bulbar involvement. L123 Drug induced MG is D-penicillamine, with onset average 6 months after initi ation of therapy. Approximately 80% of patients will remit completely within 6-8 month of cessation. L124 The Purkinje effect is a benign condition. It is a real, subjective test an d can rarely be used to determine whether a patient's retina is attached. L125 Binocular diplopia is relieved by covering one eye. Monocular diplopia is r elieved by covering the affected eye. It can be caused by keratoconus, lenticonus, high astigmatism, retinal pathologies. L126 The following results in anisocoria in dim light. a. Horner's syndrome. b. Posterior synechiae. c. Physiologic anisocoria. d. Pharmacologic anisocoria. L127 There are a host of systemic conditions that can become exacerbated during pregnancy. Pseudotumor cerebri is

not one of them. L128 AAO. PTC is associated with endocrine or metabolic dysfunction, pregnancy, or the use of exogenous substances such as vitamin A (> 100,000 U/day), tetracycline, nalidixic acid, cyclosporine, and oral contraceptives, as well as the use of or withdrawal from corticosteroids. L129 Duane. Idiopathic intracranial hypertension (IIH) is a syndrome of increase d intracranial pressure characterized by normal brain imaging, normal cerebrospinal fluid (CSF) composition, and elevated CSF pressure. IIH occurs at the same rate in pregnant and nonpregnant women, but one study showed worsening of s ymptoms among nine of 11 patients with pre-existing IIH who became pregnant. L130 Sj.gren syndrome associated with CRAO. L131 Craniopharyngioma is the most common cause of see-saw nystagmus. Other para sellar tumour can cause the condition as well. L132 Wyburn-Mason syndrome is a sporadic condition, characterized by a retinal r acemose angioma with an intracranial arteriovenous malformation (AVM). Patients with Wyburn-Mason syndrome can have o rbital AVMs as well (and consequent ocular bruits). As of 2004, there have been no reports of photodynami c therapy (PDT) to treat WyburnMason syndrome (and it is unlikely that PDT would be effective in patients with this condition). L133 Binasal hemianopia is almost never due to chiasmal compression and is usual ly due to glaucoma. B. . OCULOPLASTY OCULOPLASTY L1 Prominent pain in orbital disorder is present in orbital pseudotumour and mal ignant mixed tumour of the lacrimal gland. If pain is present, Graves' disease is usually not the cause of orbitopathy. L2 Exorbitism refers to an angle between the lateral orbital walls that is great er than 90°, which is usually associated with shallow orbital depth. This condition contrasts with telorbitism (hypertelorism) , which refers to a wider-than normal separation of the medial orbital walls. L3 The weakest orbital wall is the anterior portion of the medial wall is also k nown as the lamina papyracea, or paper-thin layer. L4 Sinus system aerates first is ethmoid. L5 The upper limit of normal exophthalmometry in white man is 22 mm, while in bl ack is 25 mm.

L6 Children's orbits are more plastic, so disorders with a short duration can ca use orbital enlargement. This is not the case in adults which needs chronic process. L7 In pediatric autoimmune hyperthyroidism, the systemic findings are more promi nent and the orbital findings less common and less severe. Furthermore, for similar degrees of exophthalmos, childr en seem to develop exposure problems less often. L8 Ocular ultrasound is usually performed in the range of 8 to 15 MHz. Higher fr equencies give better resolution. Lower frequencies give better penetration. L9 Computed tomography CT uses less radiation than conventional tomography. L10 Orbital mass. a. Sinus mucocele will have round, extraconal, smooth regular border, and very l ow internal reflectivity. b. Lymphangioma will have irregular border. c. Neurofibroma will have high internal reflectivity. d. Metastatic cancer and pseudotumour will have irregular border and high reflec tivity. e. Cavernous haemangioma has densely packed vascular channel leads to high inter nal reflectivity. L11 The magnetic field strength used to generate MRI is 2000-15000 times that of the earth's magnetic field. L12 If fat brighter it is T1 MRI, if vitreous is brighter it is T2. L13 A bluish bulge above the medial canthus is typically meningocele, whereas on e below is typically a dacryocele. L14 In infant, increases risk of spread of preseptal cellulitis to orbital cellu litis, so that; infant with preseptal cellulitis should be treated immediately with IV antibiotic. Inappropriate or delayed treat ment increases the risk. L15 The agent most likely to cause a severe preseptal cellulitis leading to seco ndary orbital cellulitis and central nervous system (CNS) infection in infants and toddlers is Haemophilus influenzae. Intrav enous antibiotics are indicated for Haemophilus influenzae cellulitis in children. The Hib vaccine has significantly reduced the incidence of preseptal and orbital cellulitis secondary to H. influenzae. L16 Staphylococcus aureus is the most common cause of preseptal cellulitis secon dary to trauma. Streptococcus organisms are a close second. L17 90% of orbital cellulitis is secondary to paranasal sinus extension. L18 Complication of orbital cellulitis is cavernous sinus thrombosis characteris ed by sudden worsening of orbital cellulitis with virtually frozen orbit, despite inapparent increase in proptosis, corneal s ensation is likewise diminished, but VA remain grossly intact. L19 Tests that are important for distinguishing infectious orbital cellulitis fr om inflammatory orbital pseudotumor include a. Oral temperature. b. Orbital computed tomography (CT) scan. c. Complete blood count (CBC) with differential. L20 Patients with orbital cellulitis are typically febrile. For pseudotumor, thi s is uncommon except in children. A leukocytosis with left-shift is also more common in cellulitis. The sedimentation rate may be elevated in either condition, and in both, computed tomography (CT) scan may show a nonspecific ret robulbar infiltrate but may have signs specific to one of the two disorders (e.g., muscle enlargement, subperiost

eal abscess). L21 Orbital cellulitis: failure to dramatically improve after 48-72 hour of anti biotic can indicate subperiosteal abscess. Relapse after switching to oral antibiotic also may be a sign. Inappropriate cho ice of antibiotic should not be associated with modest improvement. L22 Dermoid cyst may induce bony erosion on radiography. The variety that is sil ent until adulthood is generally intraorbital (retroseptal). L23 Teratomas. a. Teratomas are rare tumors that arise from two or more germinal layers (but al l three need not to be), including ectoderm and endoderm or mesoderm (or both). b. These tumors are usually cystic and can cause dramatic proptosis at birth. c. Exenteration is sometimes performed because of the fear of malignancy. Howeve r, in some cases, cystic teratomas can be removed without an exenteration, and ocular function may be pre served. d. Orbital teratomas may simulate malignancy but rarely, if ever, metastasize. L24 Potentially important complications of capillary hemangiomas in childhood ar e occlusion amblyopia, significant astigmatism. The third is significant cosmetic deformity. L25 More than 90% of periocular capillary haemangioma manifest by 6-8 months of age. Greater than 50% by 1-2 month of age. Reach their peak at approximately 6-12 month of age. L26 . Nevus flammeus is part of Sturge-Weber syndrome and Klippel-Trenaunay synd rome Nevus flammeus is part of Sturge-Weber syndrome and Klippel-Trenaunay syndrome L27 Capillary haemangioma will blanch with pressure whereas the naevus flammeus does not. L28 Neither capillary nor cavernous hemangiomas have high blood flow, and neithe r metastasize. Cavernous hemangiomas are rarely seen in youth, whereas, as the name of each implies, the blood-filled spaces are tiny in a capillary hemangioma and large in a cavernous hemangioma. L29 Thrombocytopenia in association with visceral capillary haemangioma is known as Kasabach-Merritt syndrome.

L30 lack fibrous capsule so that its growth is infiltrative. Lymphoid tissue, ev en follicles may be found Lymphangioma Lymphangiomawithin the substance of the tumour. Unlike capillary haemangioma the cystic spaces are large. L31 The origin of orbital lymphangioma remains unclear, although vascular malfor mations with lymphatic and venous components may play a role. Furthermore, noncontiguous vascular malformations ha ve been reported in one study to occur in up to 25% of patients. Surgical strategies are frequently limited to deb ulking by any safe means possible, because complete excision is usually not possible. L32 Tram-tracking of the optic nerve is considered classic for optic nerve sheat h meningioma. L33 A radiographic feature is considered pathognomonic for optic nerve glioma is kinking of the optic nerve. L34 Optic nerve glioma may induce a secondary meningothelial hyperplastic respon se misinterpreted as meningioma. L35 The management of optic nerve glioma in children is a subject of much discus sion and honest disagreement among the experts. Any of the options may be correct in a given situation, and the ult imate choice must be individualized to the patient's circumstances (e.g., vision, size of tumor, extent of centripetal spre ad, age). L36 arise from undifferentiated pluripotential mesenchymal elements in the orbit al soft tissues Rhabdomyosarcomas Rhabdomyosarcomasand not from the extraocular muscles. They may be grouped into four categories: a. . This is by far the most common type found in the orbits of infants and chil dren, accounting for Embryonal Embryonalover 80% of cases. The embryonal form has a predilection for the supero nasal quadrant of the orbit. Embryonal rhabdomyosarcomas are associated with a good (94%) survival rate. b. . This form has a predilection for the inferior orbit and accounts for 9% of orbital Alveolar Alveolarrhabdomyosarcomas. This is the most malignant form of rhabdomyosarcoma. c. . Pleomorphic rhabdomyosarcoma is the least common and the most differentiate d form. The Pleomorphic Pleomorphicpleomorphic variety tends to occur in older persons and has the best prognosis (97% survival rate). d. . This rare variant of embryonal rhabdomyosarcoma appears grapelike. It is no t found in the orbit Botryoid Botryoidas a primary tumor; rather, the botryoid variant occurs as a secondary i nvader from the paranasal sinuses or from the conjunctiva. e. Metastatic workup includes lumbar puncture and bone marrow biopsy, best done under anesthesia. L37 Duane. Although no single regimen is appropriate for every child with orbita l rhabdomyosarcoma, a sample protocol might include multiple 3-week cycles of chemotherapy, each beginning with intrav enous vincristine, actinomycin-D, and cyclophosphamide, with vincristine repeated on the 8th and 15th days of each cyc le. The regimen might include external radiation to a total dosage of 5,040 cGy. For poor prognosis cases (e.g ., metastatic alveolar rhabdomyosarcoma), newer agents under investigation include ifosfamide, etoposid e, irinotecan, topotecan, and tirapazamine. Having made the diagnosis and contributed to local staging at the time of presentation, the orbital

surgeon continues to follow the patient along with the pediatric oncology team. In cases of treatment failure, salvage surgery may take the form of orbital exenteration or excision of residual tumor combined with brachytherapy. Rhabdomyosarcoma underscores the importance of clinical suspicion when dealing w ith acute proptosis in childhood. Prompt referral to a tertiary center after appropriate imaging is the responsibi lity of the primary ophthalmologist, family practitioner, or pediatrician who first encounters the patient. L38 Most periocular metastases in children are orbital, unlike in adults. The mo st common is neuroblastoma. L39 Fungal species likely leads to a necrotizing orbital cellulitis are Mucor, R hizopus, and Aspergillus. L40 is generally seen only in the immunocompromised (chemotherapy, posttransplan t, diabetes Mucormycosis Mucormycosismellitus). Aspergillosis is probably more common in this population also, but it may be seen in otherwise healthy individuals, and it also may be seen as a result of allergic aspergillosis sinus itis. L41 Orbital phycomycosis generally results from invasion by necrotizing fungal s inusitis. Black eschar in the nasal cavity is virtually diagnostic but is a late finding. (Its absence does not exclude the diagnosis.) L42 Sebaceous cell carcinoma is more common in women, while fibrous histiocytoma are more common in men. L43 Thyroid-related orbitopathy can occur despite a persistently euthyroid state (as indicated by clinical and laboratory findings). L44 The indications of immediate surgical intervention in patients with Graves's orbitopathy are acute corneal decompensation and evidence of optic nerve compression. L45 Sensitive immunometric assay. This is the most sensitive way to thyroid-stimu lating hormone (TSH) levels thyroid-stimulating hormone (TSH) levelsdetect hyperthyroidism; feedback regulat ion can lead to a reliably detectable decrease in thyroid-stimulating hormone (TSH), even when fluctuating levels of thyroxine (T4) and 3, 5, 3'-triiodothyron ine (T3) may not be indicative.

L46 Surgical procedures that may be indicated during the active, inflammatory ph ase of Graves' ophthalmopathy include: a. Lateral tarsorrhaphy. b. Orbital decompression. L47 The histopathologic changes evident in extraocular muscle (EOM) specimen fro m patients with Graves' ophthalmopathy also may be seen in the lacrimal glands. Fibrosis tends to be les s severe, leading to partial loss of the normal glandular tissue.

L48 Typical manifestations of idiopathic orbital inflammation (pseudotumor) incl ude all of the following: a. Dacryoadenitis. b. Extraocular myositis. c. Periscleritis. d. Optic perineuritis. e. Changes in corneal sensation may occur, but frank ulceration especially periphe rally is unusual L49 Clinical findings that are more likely in pediatric orbital pseudotumor than in the adult variety of the disease include all of the following: a. Bilateral involvement. b. Systemic symptoms and signs (malaise, fever, vomiting). c. Peripheral eosinophilia. d. Uveitis. L50 Orbital pseudotumor is generally quite painful for both children and adults. L51 Some patients do not experience the typical pain. Some may have minimal infl ammatory signs and may present with a totally fibrotic scarred lesion (idiopathic sclerosing inflammation of the orb it). Such lesions may require biopsy for diagnosis because it is misdiagnosis for malignancy. There is little response to steroid because it is fibrotic. L52 Orbital pseudotumor. Although a dosage of 60 mg of prednisone is at the lowe r end of the usual beginning ranges for steroid therapy, one should consider an orbital biopsy because of the possib ility of other inflammatory diseases and other orbital pathology. After the diagnosis is confirmed, additional attempts s hould be made to bring about a response with stronger steroid therapy, orbital radiation, methotrexate, or cyclophospham ide. L53 Unlike Graves' ophthalmopathy, pseudotumor can and commonly does involve the muscle tendons as well as their insertions, along with posterior Tenon fascia. L54 Sarcoidosis generally not associated with pain and spares orbital soft tissu e. L55 The following disorders may be associated with a clinical presentation indis tinguishable from typical inflammatory orbital pseudotumor. a. Systemic lupus erythematosus (SLE). b. Polyarteritis nodosa. c. Wegener's granulomatosis. d. Churg-Strauss syndrome. L56 Bilateral painless enlargement of the lacrimal glands is typical presentatio n of sarcoidosis and benign lymphoepithelial lesion. L57 Hemangiopericytomas are more likely to limit ocular motility than cavernous hemangiomas. L58 Hemangiopericytomas (rapidly expansile and aggressive tumour) may restrict o cular motility and cause conjunctival prolapse and engorgement of vessels in the conjunctival cul-de-sac. Hemangiopericytomas resemble cavernous hemangiomas on contrast-enhanced CT and may look blue in surgery. Thes e lesions must be completely excised because they may recur, undergo malignant degeneration, and metastasize. Histopathologically, these lesions are unique in that microscopically "benign" lesions may recur and metast asize, whereas microscopically "malignant" lesions may remain localized.

L59 Most orbital meningiomas arise outside the orbit and invade secondarily. L60 In NF, the growth pattern of meningioma tends to be paraxial (diffusely alon g the nerve sheath) rather than focal. L61 Meningioma may be associated with bony changes either osteolytic or osteobla stic, so that CT is a modality of choice in the initial evaluation. Angiography generally reveals a highly vascula rized tumour (tumour blush). MRI is particularly useful for the suspected intracranial extension. L62 Optic nerve sheath meningioma. Excisional biopsy of optic nerve biopsy gener ally result in visual loss, because fragile pial blood vessel surrounding tumour. Surgeries are usually indicated if there is intracranial extension, severe proptosis and sever visual loss. Also surgery is usually indicated if tumour gro wth is becoming a potentially a life threatening. Paediatric meningioma is more aggressive tumour than adult variety and is removed earlier. L63 Ophthalmologist specialist practice 10% of lacrimal gland lesion will be inf lammatory or lymphoid. In contrast, to orbit specialist practice probably see 50% epithelial neoplasm and 50% inflammatory/ly mphoid lesions. L64 The critical features of lacrimal disease: a. Pain is more typical of acute inflammatory, infectious, or malignant lesions. b. Duration of symptoms is helpful in separating slowly progressive benign lesio n, such as benign mixed tumour from more acute conditions such as malignant neoplasm or inflammation. c. Radiological findings osteolysis is strongly favor malignant lesion. L65 Pain in adenoid cystic carcinoma is more commonly due to perineural extensio n and bone destruction. L66 Soft-tissue contour analysis can help differentiate between lymphoid tumors (elongated smooth masses pancakes ) from parenchymal tumors (globular masses). Adenoid cystic carcinomas usu ally produce bony destruction. Benign mixed cell tumors often cause concave pressure changes in the adjacent bo ne. Lymphoid and inflammatory lesions generally do not cause bony changes. L67 Benign mixed cell tumor, the male-female ratio is 3:2. The tumor should be a pproached through a lateral orbitotomy with careful excision to avoid rupture of the tumor's pseudocapsule. Incisional or incomplete biopsy techniques can lead to infiltrative tumor recurrence (in 32% of cases) and, occasionally, malignant transformation.

L68 Histiocytic disorders is most likely to involve orbital bone is eosinophilic granuloma. L69 Fibrous histiocytoma is one of the most common mesenchymal tumours of the or bit. 10% have metastatic potential. Another 16% are termed locally aggressive but not frankly malignant. Most are benign (>90%), and have a storiform, or matlike, pattern on histopathology. It is usually very firm and ca n displace other orbital structures. L70 Fibrous dysplasia may be monostotic (affect only one bone) or polyostotic. T he polyostotic variety may present with precocious puberty and dermal hyperpigmented macules. This disorder, Albright's syndrome, rarely involves the orbit. Orbital disease is nearly always monostotic and rarely associated with precociou s puberty, regardless of age. Surgical curettage or excision is usually undertaken. Only woven bone, not cancellous, is found microscopically. L71 It is impossible to differentiate clinically or radiographically between ben ign reactive lymphoid hyperplasia It is impossible to differentiate clinically or radiographically between benign reactive lymphoid hyperplasia and orbital lymphoma. Biopsy with light microscopy, immunochemical staining, and electron microscopy is and orbital lymphoma. Biopsy with light microscopy, immunochemical staining, and electron microscopy is . necessary to distinguish between them necessary to distinguish between them L72 The central nervous system (CNS) is not routinely surveyed in patients with orbital lymphoma. This is in The central nervous system (CNS) is not routinely surveyed in patients with orbi tal lymphoma. This is in contrast to patients with intraocular lymphoma. When CNS involvement is suspecte d, computed tomography contrast to patients with intraocular lymphoma. When CNS involvement is suspecte d, computed tomography . (CT) scan or magnetic resonance imaging (MRI) is the starting point (CT) scan or magnetic resonance imaging (MRI) is the starting point L73 The key features separating pseudotumour from lymphoproliferative lesion are : a. Prominent fibrovascular stroma. b. Hypocellularity. L74 The three orbital deposits that are hyperintense on T1 weighted MRI are bloo d, melanin, and mucus. L75 Exceptions to the rule of thumb that all orbital tumors are dark on T1-weigh ted magnetic resonance images (MRI) include: a. Retrobulbar hemorrhage of at least 24 hours duration. b. Melanoma. c. Mucocele. L76 The nose and paranasal sinuses are more frequently the focus of tumors that secondarily invade the orbit. L77 The most common sinus lesion that invades the orbit is the mucocele. The mos t common sinus neoplasm to invade the orbit is squamous cell carcinoma. L78 Classic features of a tripod fracture include all of the following: a. Downward displacement of the lateral canthus. b. Infraorbital hypesthesia. c. Trismus. d. Temporal subconjunctival hemorrhage. L79 The tripod complex is produced by three distinct fractures along suture line

s zygomaticofrontal, zygomaticomaxillary, and the zygomatic arch. Ocular motility may or may not be n ormal in pure tripod fractures, but upgaze should be spared. Repair is indicated when there is marked cosmetic defor mity or potential mandibular instability. L80 Telecanthus and rounding of the medial canthus is a characteristic finding i n direct naso-orbital-ethmoidal fractures. L81 The qualifier indirect implies that forces other than direct contact with a bl unt object led to orbital wall fracture (for instance, suddenly increasing intraorbital pressure). This type is rarely associ ated with orbital rim fracture, which is seen more. L82 The following findings in orbital floor fractures is/are exacerbated by the presence of a coincident medial orbital wall fracture are enophthalmos, subcutaneous emphysema. L83 With severe muscle contusion, oedema, or haemorrhage, forced duction test ma y be falsely positive or impossible to interpret. They are more fruitful if performed 5-10 days after injury. L84 Most diplopia associated with orbital contusion disappears 7-14 days followi ng injury. Most diplopia result as reversible muscle contusion. L85 The following are considered indications for surgical repair of orbital floo r fractures: a. Disabling diplopia present 7 to 10 days after the original injury. b. Large fracture on acute computed tomography (CT) scans. c. Enophthalmos >2 mm. d. Inferior rectus entrapment in a pediatric patient present 1 day after injury. L86 Computed tomography (CT) scan of the orbit with direct coronal, axial, and s agittal views gives better definition of the soft tissue and bony structures than plain films. Magnetic resonance imaging (MRI) has a very limited role, because bone is dark. L87 Large, complex anterior fractures lead to marked inferior orbital herniation without entrapment. This causes Large, complex anterior fractures lead to marked inferior orbital herniation wit hout entrapment. This causes hypoglobus and enophthalmos. Small posterior fracture can cause significant entr apment as a crowded hypoglobus and enophthalmos. Small posterior fracture can cause significant entr apment as a crowded . muscle belly is forced or pinched into the defect. In these cases, enophthalmo s is minimal muscle belly is forced or pinched into the defect. In these cases, enophthalmos is minimal L88 : Indication of removal of intraorbital FB Indication of removal of intraorbital FB a. Vegetable FB.

b. Anterior, easily approachable FB. c. When there is evidence of optic nerve compromise as decreased VA, APD, and dy schromatopsia. L89 The most common postoperative complication of enucleation is superior sulcus deformity. L90 When ocular or optic disc perfusion is severely compromised by an orbital co mpartment syndrome, immediate canthotomy with cantholysis should be performed to decompress the orbit. L91 In direct optic nerve trauma, something physically impinges on the optic ner ve, but it need not be a foreign body (e.g., bone fragments). In indirect trauma, nothing can be found to have directl y struck the nerve. L92 Treatment of indirect optic nerve injury is most likely with emergent high-d ose intravenous methylprednisolone to reduce swelling associated with the injury. Optic canal decompression may also b e considered. L93 Risk factors that increases the chance of extrusion of orbital implants: a. Implant too large for the orbit. b. Conformer too large for the fornix. c. Exposed synthetic integrated implant. d. Orbital tissue infection. L94 The procedure of choice for severe socket contracture following enucleation involves the use buccal mucosa grafts. L95 Whitnall's ligament should not be confused with the horns of the levator apo neurosis, which are structure to be cut during ptosis surgery. L96 The levator palpebrae superioris, the deeper or posterior portion inserts on to the anterior surface of the lower half of the tarsus, not to its superior border. Müller's muscle inserts into the superior border of the tarsus. L97 Following trauma to the medial canthus, special attention must be directed t o reattaching the tendon to its more posterior attachment, the posterior lacrimal crest. L98 The normal mongoloid slant of the palpebral fissure occurs because of the high er insertion of the lateral canthal tendon (3 mm higher than the medial canthal tendon [MCT]). Antimongoloid slant occ urs when the lateral tendon inserts lower than the medial and can occur with trauma (e.g., tripod fracture) or congenital abnormalities (e.g., Treacher Collins' syndrome). L99 Lower eyelid ectropion is a variable feature not a defining feature of bleph arophimosis syndrome. L100 Blepharophimosis is also known as Kohn-Ramono syndrome. L101 Primary amenorrhea is the most common condition associated with blepharophi mosis syndrome. L102 Congenital coloboma of the upper eyelid is generally isolated whereas that of the lower eyelid is more commonly associated with other facial abnormalities. L103 The most likely outcome following inadvertent suturing of the orbital septu m into subcutaneous tissue when repairing a partial thickness eyelid laceration is lid retraction in downgaze. L104 Hairless skin of similar pigmentation must be chosen for free skin graft in the setting of lid laceration. L105 Factors important to evaluate before planning therapy for involutional ectr opion are: a. Position of the inferior punctum. b. Stability of the lower limb of the MCT. c. Stability of the lateral canthal tendon. d. Presence or absence of the contracture of the orbicularis muscle.

L106 Entropion of any aetiology is more common in the lower lid. L107 Digital pressure along the inferior border of the inferior tarsus will temp orarily correct involutional entropion Digital pressure along the inferior border of the inferior tarsus will temporari ly correct involutional entropion but not cicatricial entropion. but not cicatricial entropion. In cicatricial entropion, vertical foreshortening is present, rather than the In cicatricial entropion, vertical foreshortening is present, rather than the . redundancy of involutional entropion redundancy of involutional entropion L108 To allow maximal spontaneous return of function before surgical repair, it is generally wise to observe traumatic ptosis in an adult for 6 months. Ptosis repair before the possible spontaneous r eturn of function may lead to lagophthalmos. L109 Lid notching, scarring, hypopigmentation, and edema may be seen after lash cryoepilation. L110 Critical components in the evaluation of corneal protection in patient with ptosis surgery: a. Assessment of lagophthalmos. b. Assessment of Bell phenomenon. c. Schirmer testing. d. Assessment of corneal sensation. L111 The primary abnormality seen in ptosis after cataract surgery is in the lev ator aponeurosis. Aponeurotic dehiscence has been blamed on anesthetic injection, lid specula, and bridle sutures. L112 Ptosis of MG is often resistant to anticholinesterase or steroid. Surgery i s usually performed after medical therapy has been optimized. L113 In Graves ' disease lateral lid retraction is greater than medial, in conve rse to the collier sign. L114 Recession of the superior rectus muscle will induce lid retraction, while r esection will induce ptosis. L115 Lower lid retraction is more challenging than the upper lid repair in patie nt with Graves ' disease, because it needs spacer graft.

L116 Acanthosis nigricans: suddenly appearing multiple seborrheic keratoses know n as Leser-Trelat sign is Acanthosis nigricans: suddenly appearing multiple seborrheic keratoses known as Leser-Trelat sign is . usually associated with GIT malignancy usually associated with GIT malignancy L117 Eyelid lesions are typically slightly elevated with a central ulcerated are a or crater are: a. Molluscum lesion. b. Basal cell carcinoma. c. Keratoacanthoma. L118 Cryotherapy selectively destroys melanocytes but is insufficient for cutane ous melanoma and should be considered a palliative treatment. L119 Blepharochalasis is a rare idiopathic disorder leading to inflammatory edem a of the eyelids. It is familial, and younger patients, especially women are affected. The repeated episodes of edema may caus e ptosis and herniation of the orbital lobe of the lacrimal gland. Dermatochalasis is redundant preseptal skin caused by aging. True ptosis (involutional) may be present in either disorder. L120 Hemifacial spasm is rarely bilateral and is usually caused by vascular comp ression of the seventh cranial nerve at the brainstem and can result in synchronous contractions of the entire side of t he face. Along with partial complex seizures and myoclonic epilepsy, essential blepharospasm is effaced by sleep. L121 Any treatment for blepharospasm is designed to decrease eyelid closure. Thu s, dry eye will be aggravated. Facial nerve ablation suffers from recurrence rates as high as 30% and is associated wi th complications such as hemifacial paralysis. Consequently, its use has significantly decreased. Orbicularis myecto my is usually preferred to facial nerve avulsion. Botulinum injection is the initial treatment of choice. L122 Retrospective studies show that the mortality rate from ocular adnexal basa l cell carcinoma is 3%. The vast majority of patients who died from basal cell carcinoma had disease that started in the c anthal areas, had undergone prior radiation therapy, or had clinically neglected tumors. L123 Removal of the orbital lacrimal gland removes the efferent input and interf eres with reflex tearing, whereas removal of the palpebral lobe damages the ducts from the orbital portion, which run thro ugh the palpebral lobe. This impairs reflex tearing as well. L124 Ganglia serves as the home for cell bodies providing postganglionic efferen t innervation to the lacrimal gland is the sphenopalatine ganglion which receives parasympathetic fibers from the greater s uperficial petrosal nerve (a division of CN VII). L125 In congenital NLD obstruction, the blockage is at the valve of Hasner. In a cquired cases, the blockage is within Intraosseous NLD. Involutional stenosis is one of the most common causes of acqu ired NLD obstruction. L126 Jones 1 test, one third of normal patient will have false positive result w ith this test. L127 Single functioning punctum and canaliculus is usually sufficient. L128 Probing of acquired obstructions rarely yields permanent patency. L129 Congenital canalicular obstruction is rare in infants. L130 Acquired causes of canalicular obstruction. a. Trauma.

b. Phospholine iodide. c. Actinomyces infection. d. Idoxuridine use. e. Contraceptive pill. f. Steven-Johnson syndrome. L131 Congenital obstruction of the nasolacrimal system, general anesthesia is ge nerally necessary after the age of 6 months. L132 Mortality rate of sebaceous gland carcinoma is approximately 20%, although variable studies have published mortality rate from 3% to 41%. L133 Acute, lancinating pain in the medial canthal region with minimal noninflam ed enlargement of the lacrimal sac is most subjective of impacted dacryolith. L134 The most common malignant tumor of the lacrimal sac is squamous cell carcin oma. Treatment may consist of a dacryocystectomy and a rhinotomy. L135 The following constitute indications for dacryocystorhinostomy (DCR). a. Recurrent acute dacryocystitis. b. Chronic discharge or symptomatic epiphora with a positive secondary dye test (organic obstruction). c. Persistent epiphora in a child after probing and Silastic intubation of a con genitally impatent system. L136 In probing the nasolacrimal system of an infant with congenital stenosis, i t is better to start with the superior canaliculus. The superior canaliculus possesses more maneuverable angles and hen ce is easier to probe. Furthermore, there is less functional significance if a false passage is created. L137 The average onset of action of botulinum toxin is 48 to 72 hours. Peak botu linum effect usually occurs 2 to 3 weeks after injection. L138 Any patient with sclerokeratitis and coexisting sinus disease should be sus pected for Wegener's granulomatosis. Ocular disease occurs in 60% of patients with the disease. Eighty percent of pat ients are positive for antineutrophilic

cytoplasmic antibodies, or cytoplasmic pattern (c-ANCA). The use of immunosuppre ssive drugs (especially cyclophosphamide) has significantly improved the mortality rates of patients wit h Wegener's granulomatosis. L139 Lymphangioma is not encapsulated, while schwannoma, cavernous haemangioma, and hemangiopericytoma are all encapsulated. L140 Schwannoma, an encapsulated tumour displaying the classic Antoni-A spindle cells and nuclear palisading (Verocay bodies). Schwannomas rarely undergo malignant transformation. They can recur aft er incomplete excision. L141 The neurilemoma (also called schwannoma) arises from Schwann cells. Slow gr owing and encapsulated, this yellowish tumor may show cysts and areas of hemorrhagic necrosis. It may be soli tary or associated with neurofibromatosis. Two histologic patterns appear microscopically: Antoni-A spin dle cells are arranged in interlacing cords, whorls, or palisades that may form Verocay bodies, or collections of fibr ils resembling sensory corpuscles. Antoni-B tissue is made up of stellate cells with a mucoid stroma. Vessels are u sually prominent and thick walled, and no axons are present. L142 Schwannomas, sometimes known as neurilemomas, are proliferations of Schwann cells that are encapsulated by perineurium. These tumors have a characteristic biphasic pattern of solid areas with nuclear palisading (Antoni A pattern) and myxoid areas (Antoni B pattern). Hypercellular schwannomas sometime s recur even after what is thought to be complete removal, but they seldom undergo malignant transformation. These tumors are usually well encapsulated and can be excised with relative ease. L143 Ocular stinging/burning is the most common side effect of topical cyclospor ine occurs in 15% of cases. L144 Topical cyclosporine is contraindicated in patient with active infection as active blepharitis. L145 The treatment of choice for patient with localized orbital lymphoproliferat ive lesion is radiation. Neither surgical excision nor systemic steroid is recommended. L146 Lesions may be associated with proptosis with the Valsalva maneuver or cryi ng. a. Orbital lymphangioma. b. Capillary hemangioma. c. Orbital varix. d. Encephalocele (outside lamkin). L147 Lymphangiomas. a. Lymphangiomas are primarily a disorder of the pediatric age range. b. Superficial lesions may have a bluish or violaceous hue. c. Classic presenting symptoms include spontaneous ecchymosis, and proptosis wit h crying or following upper respiratory infections. C. . UVEITIS UVEITIS L1 Passive immunization (parenteral administration of antibody) confers immediat e but short lived protection, whereas active immunization (vaccination with active or inactivated immunogens) confers protection that lasts months to years. L2 Typically, lipids and nucleic acids are not antigenic but may become so if co upled with proteins or polysaccharides. L3 IgM or IgM-like immunoglobulins tend to be the only type present in organisms

with the most rudimentary immune systems (relative to mammals). IgM immunoglobulin class is probably the oldest p hylogenetically. L4 The cyclooxygenase pathway leads to the production of prostaglandins, thrombo xane, and prostacyclins, whereas the lipoxygenase pathway leads to the production of leukotrienes. L5 Up to 90% of patients with ankylosing spondylitis are positive for HLA-B27. H LA-B27 is also associated with several other diseases, although the chance that an HLA-B27-positive individual will hav e a seronegative spondyloarthropathy or eye disease is approximately 25%. Note that 85% to 95% of patients with Reite r's syndrome are HLA-B27-positive as well. Fifty percent of patients with acute iritis may be HLA-B27-positive. L6 Dalen-Fuchs nodules are focal accumulations of epithelioid like cells between Bruch's membrane and the retinal pigment epithelium (RPE). They may include depigmented RPE cells. They are class ically associated with sympathetic ophthalmia (SO) and Vogt-Koyanagi-Harada (VKH) syndrome. They also may be found in tuberculous choroiditis and sarcoidosis. L7 Idiopathic iridocyclitis is the most common cause of anterior uveitis, making up at least 10% of all uveitis cases. HLAB27 iridocyclitis is the second most common cause, and juvenile rheumatoid arthr itis (JRA) and herpes (simplex or zoster) follow in incidence. L8 Toxoplasmosis is the most common cause of posterior uveitis, accounting for u p to 7.0% of total uveitis cases. Other causes of posterior uveitis include retinal vasculitis, necrotizing herpetic ret inopathy, and idiopathic causes. L9 The differential diagnosis of diffusely distributed keratic precipitates incl udes Fuchs' heterochromic iridocyclitis, and rarely sarcoidosis, syphilis, and toxoplasmosis. The diffuse distribution, a long with a gelatinous, stellate appearance, makes the keratic precipitates (KPs) of Fuchs' iridocyclitis distinc tive. L10 HLA-B27 is present in 1.4% to 6% of the general population. L11 Ritonavir is a protease inhibitor which can increase CD8+ lymphocyte counts. L12 The following conditions are associated with the HLA-B27 genotype. a. Inflammatory bowel disease. b. Reiter's syndrome. c. Ankylosing spondylitis.

d. Psoriatic arthritis. L13 Up to 25% of individuals with HLA-B27 develop sacroiliac disease. Symptoms o f sacroiliac disease may be subtle. Personal or family history of back problems in patients with iritis should promp t the physician to obtain sacroiliac radiographs. L14 Asymptomatic sacroiliac disease can be seen in patients with HLA-B27 spondyl itis, particularly in young men. Because irreversible damage may occur before the onset of significant symptoms a nd simple physical therapy is effective in limiting disability, physical therapy, consisting of back flexibili ty and stretching exercises, is recommended in young men who are found to be HLA-B27-positive. L15 Although it is associated with both forms of inflammatory bowel disease, iri tis occurs more commonly in patients with ulcerative colitis. Less than 15% of patients with ulcerative colitis and 10%. (Earlier series documented rates approaching 40%, but these studies defined virtually every fundus finding as endophthalmitis, including nonspecific retinal vascular changes such as Roth's spots and nerve-fiber layer infarcts. The distinction is important because chori oretinal involvement without vitreous infection will respond to system antifungals, obviating the need for vitrectomy and/or intravitreal antibiotics. The reason for the lack of vitreous involvement is presumably the response to systemic anti fungal agents). L54 In a recent study, the risk of future multiple sclerosis (MS) following a di agnosis of pars planitis exceeds 16%. The risk was significantly greater among patients presenting with retinal vascular s heathing. L55 Clinical features distinguishing the progressive outer retinal necrosis (POR N) syndrome from the acute retinal necrosis (ARN) syndrome include: a. Initial involvement of the outer retina. b. Relative lack of intraocular inflammation. c. Relative lack of vasculitis. d. Earlier involvement of the macula. L56 Both progressive outer retinal necrosis (PORN) and acute retinal necrosis (A RN) are manifestations of severe posterior segment infection by the herpes family of viruses. ARN, particularly i n immunocompetent hosts, responds to intravenous acyclovir more reliably than PORN. Up to two thirds of PORN eyes wil l end up with no light perception (NLP), despite treatment. L57 One potential complication of cryotherapy for pars planitis is rhegmatogenou s retinal detachment. Laser photocoagulation may have the same salutary effect on resistant pars planitis as cryotherapy, but without the risk of retinal detachment (RD). The following are therapeutic options for pars planitis . (Treatment is usually started if cystoid macular edema (CME) is evident, or if visual acuity is 14mm and onset 50%. A 60% reduction in progression t o severe visual loss in patients with socalled high-risk characteristics was reported. L12 Disc neovascularization (NVD) greater than one fourth to one third of a disc area is considered high risk, whether or not it is associated with vitreous hemorrhage. This is in distinction to neovasc ularization elsewhere (NVE), which must be associated with bleeding to qualify as high risk. Furthermore, to meet the Di abetic Retinopathy Study (DRS) criteria for high-risk disease, NVE also must comprise at least half the disc area. Any N VD associated with hemorrhage is high risk. L13 Adverse effects of PRP include: a. Decreased night vision results from destruction of extramacular rods. b. Angle-closure glaucoma may occur after particularly heavy treatment associate d with choroidal effusions. c. Regression of neovascular fronds may be associated with contracture and secon dary rhegmatogenous

retinal detachment (RRD) or traction retinal detachment (TRD). d. Laser can cause temporary or permanent dysfunction of the long ciliary nerves passing through the outer choroid and cause difficulties with corneal sensation and accommodation. The lat ter probably occurs far more often than suspected. e. PRP also can aggravate macular edema, further compromising near acuity. L14 With sickle cell (SC) disease, obstruction of the central macular blood supp ly may occur due to thrombosis of a cilioretinal or macular branch artery. This complication is very unusual in othe r forms of sickle hemoglobinopathies. Conversely, neovascularization (NV) is a relatively uncommon feature of SC retin opathy. If choroidal neovascularization (CNV) occurs, it is usually related to angioid streaks. As in diabetes, contract ure of neovascular fronds may lead to retinal detachment (RD). L15 Although rare, cases of retinopathy have been reported in association with s ickle cell trait (Hb AS) only. In addition, sickle cell SC disease can be associated with angioid streaks and comma-shaped c onjunctival capillaries. The incidence of proliferative retinopathy is as follows: sickle cell SC disease (Hb SC), 33%; sickle cell thalassemia (Hb SThal), 14%; sickle cell SS disease (Hb SS), 3%. Preretinal hemorrhage may rarel y be seen in nonproliferative disease, but it typically heralds neovascularization (NV). L16 A number of systemic disorders are associated with angioid streaks, includin g pseudoxanthoma elasticum ( P ), Ehlers-Danlos syndrome ( E ), Paget's disease of bone ( P ), sickle cell (SC) disease an d SC trait ( S ), and idiopathic ( I ) causes remember these by the acronym PEPSI. Approximately 50% of cases of angioid streaks are idiopathic. Photodynamic therapy may be of some benefit in the treatment of choroidal neovas cularization (CNV) secondary to angioid streaks. L17 Modalities useful in the treatment of proliferative sickle cell (SC) retinop athy include: a. Pars plana vitrectomy with endolaser. b. Scatter photocoagulation in the region of a neovascular frond. c. Feeder-vessel photocoagulation. d. Photocoagulation to close the feeder vessels of neovascular fronds can cause regression of the neovascularization (NV) but is associated with a relatively high rate of complic ations (hemorrhage). Scatter

photocoagulation to the region involved with the NV can effectively control it, with fewer complications. With dense vitreous hemorrhage, vitrectomy may be necessary to permit laser treatment . L18 Ranibizumab (Lucentis) is a potent antiangiogenic agent administered through intravitreal injection and is derived from bevacizumab (Avastin), a monoclonal antibody to VEGF. Phase III data from G enentech's MA-RINA study demonstrated visual acuity improvement in approximately one third of patients. I t is derived from a monoclonal antibody to vascular endothelial growth factor (VEGF). L19 The development of retinopathy of prematurity (ROP). a. It is hypothesized that immature vascular precursor tissue is susceptible to oxygen-induced cytotoxicity. b. Environmental oxygen may be sufficient to induce retinopathy of prematurity ( ROP) in some cases. L20 Several retrospective studies have suggested that laser therapy is at least as effective as cryotherapy, with fewer complications and less stress on the child. Laser treatment causes significantly less soft-tissue inflammation than cryotherapy. L21 Complications of branch retinal vein occlusion (BRVO) can be divided into ac ute and chronic categories. Macular edema, macular nonperfusion, and hemorrhage may occur acutely. Macular edema, su bretinal fibrosis, and posterior neovascularization (NV) can be delayed causes of visual loss following BRVO. Rub eosis iridis occurs in 90% of cases and is associated with microphthalmia. Retinoblastoma is often bilateral and has no significant as sociations with microphthalmia or cataracts. L136 Classification of hereditary hyaloideoretinopathies into two categories has been suggested: those with ocular manifestations only (Wagner's disease) and those with systemic symptoms (Stickle r's syndrome). Wagner's disease consists of high myopia, posterior subcapsular cataract, and an optically empty vitreous. Stickler's syndrome is Wagner's disease plus increased incidence of (i) retinal detachment (RD), (ii) f acial anomalies, and (iii) musculoskeletal anomalies. L137 Familial exudative vitreoretinopathy is autosomal dominant and occurs in fu ll-term infants with normal respiratory status. L138 The following statements about asteroid hyalosis. a. It is more common with aging. b. Asteroid hyalosis is monocular in 75% of cases. c. It is generally associated with no decrease in visual acuity. d. The vitreous is otherwise normal.

e. The particulate matter seen clinically consists of calcium soaps. L139 Asteroid hyalosis usually does not settle, suggesting some collagenous supp ort of asteroid bodies within formed vitreous. Synchysis scintillans, the crystalline lipid breakdown products of pre vious hemorrhage, usually settle inferiorly in liquefied vitreous (or in an eye that has undergone vitrectomy). L140 Estimates of the proportion of vitreous hemorrhage caused by diabetic retin opathy have ranged from 39% to 54%. L141 Condition mandate(s) repair with pars plana vitrectomy rather than conventi onal scleral buckling +/- pneumatic retinopexy. a. Retinal detachment (RD) with accompanying vitreous hemorrhage following penet rating trauma. b. RD associated with marked proliferative vitreoretinopathy (PVR). c. Aphakic retinal detachment (RD) and retinal dialysis may be repaired with scl eral buckling +/- pneumatic retinopexy alone. However, traumatic RD with vitreous hemorrhage or with severe proliferative vitreoretinopathy (PVR) may require vitrectomy to remove vitreous scaffolds that can promote future neovascularization (NV) and contraction. L142 Advantages of early vitrectomy surgery in a patient who is diabetic and has vitreous hemorrhage include: a. An opportunity to remove the vitreous scaffold that fosters neovascularizatio n (NV). b. An opportunity to treat retinal ischemia intensively with endolaser. L143 Severe visual loss after vitrectomy seems more common in patients who are d iabetic. Lens removal may increase the risk of rubeosis, although a concomitant retinal detachment (RD) may be the key factor prompting lensectomy. L144 Indications for emergent pars plana vitrectomy surgery on an eye with an ac ute (i.e., unrepaired) rupture or laceration include: a. Vitreous hemorrhage and associated retinal detachment (RD). b. Intraocular foreign body. c. Traumatic endophthalmitis.

d. NB. Indications for emergent pars plana vitrectomy in the setting of globe pe netration include retinal detachment (RD), intraocular foreign body, and endophthalmitis. Vitreous hemorrh age without RD may be observed or addressed 7 to 10 days after the initial repair of globe rupture. L145 Traumatic macular hole and subfoveal hemorrhage may lead to a permanent dec rease in visual acuity. Commotio retinae and Valsalva retinopathy generally have a good visual prognosis. L146 Among men, each of the following constitutes a significant risk factor for the development of a central retinal vein occlusion (CRVO). a. Systemic hypertension. b. Diabetes mellitus. c. Open-angle glaucoma. d. Decreased physical activity. L147 According to the Eye Disease Case-Control Study, elevated erythrocyte sedim entation rate is a risk for central retinal vein occlusion (CRVO) only among women. L148 Moderate alcohol consumption reduces the risk of central retinal vein occlu sion (CRVO) for both women and men. Exogenous estrogen use also reduces risk. L149 Among patients with initially non-perfused (ischemic) central retinal vein occlusion (CRVO), the following is a risk factor for the development of at least 2 hours of iris or angle neovascularizati on (NV). a. >30 disc areas of non-perfusion. b. Male gender. c. Duration +6 sphere OU), but in this case to eliminate a near angle of 30, success i s most likely with full refractive correction. A4 Glaucoma with neurofibromatosis is seen in association with plexiform neurofi bromas of the eyelids (a secondary form of glaucoma); in the absence of a lid lesion, there is no reason to suspect that this child has elevated IOP and no reason to sedate her to measure IOP.

A5 Lisch nodules eventually develop in over 90% of patients with neurofibromatos is type I, but they are rarely present in early childhood. Lisch nodules are therefore a sensitive, but not a specific, fi nding for neurofibromatosis at this age (2 years). A6 A CT scan with coronal views, rather than MRI, is the best way to image a sub periosteal abscess. This is fortunate because a 5-year-old child usually requires deep sedation or general anesthesia for MRI but not for CT. A7 Congenital glaucoma can be mistaken by pediatricians for nasolacrimal duct ob struction, particularly if obvious corneal edema or buphthalmos are absent when the parents first complain of teari ng. A8 Upper lid lesions can cause amblyopia due to pupil occlusion. Usually, the li d must cover half or more of the pupil at all times to be significantly amblyopic. Even infants adopt head turns or chin-u p postures to avoid pupil occlusion. A9 Tucking too much or too close to the trochlea can hinder the movement of the tendon through that structure; hence, tucking generally is performed in the portion of the tendon adjacent to the nasa l border of the superior rectus muscle or more distally. A10 Absence of hyperdeviation in the lower field, where there is no occasion for the short or otherwise restricted superior oblique tendon to generate a limited rotation, is characteristic of all but the most severe cases of Brown syndrome. Divergence in straight upgaze is an important sign helping to distingu ish Brown syndrome from inferior oblique muscle palsy; it was considered by Brown to be a defining feature of thi s condition and is not associated with the opposite horizontal deviation in downgaze. A11 Exotropia in upgaze was described by Brown as an essential feature of this s yndrome, although it is not prominent in every case. Hypotropia in primary position is present in monocular elevation deficiency, but the most severe cases of Brown syndrome show this as well. A12 Possible causes of papilledema and a sixth nerve palsy are dural AVM, crania l venous thrombosis, pseudotumour and sleep apnea. Diabetes is not. A13 The restrictive changes that occur in the extraocular muscles in Graves' dis ease are preceded and accompanied by an inflammatory phase that adds a variable weakening effect to th eir action. A14 Masses presenting above the medial canthal ligament in infancy are usually d ermoid cysts or encephaloceles. It is important to rule out this last condition by imaging, as it would be a ser ious error to attempt excision of a dacryocele in this area without knowing that it does not communicate intracrania lly. Systemic antibiotics without drainage do not affect lacrimal system blockage definitively. Percutaneous incis ion and drainage too often results in fistula formation and should be avoided unless the skin is already seriously com promised. Bony stenosis of the nasolacrimal canal typically gives signs of a chronic partial obstruction rather than of an acute total closed-end obstruction. A15 Enucleation is still frequently used to treat large tumors in eyes with poor visual potential or when other methods of treatment have failed. Cryotherapy and laser therapy are usually rese

rved for small tumors, less than 2.0-2.5 mm in thickness. Plaque radiotherapy can be used as a secondary treatmen t for retinoblastoma but may be associated with a higher incidence of radiation retinopathy and optic neuropathy when used in children who have already undergone chemotherapy. A16 Steroid administration is more likely to result in cataracts or glaucoma in children than in adults. A17 The critical period for the development of deprivation begins earlier and la sts longer than that for strabismic or anisometropic amblyopia. Furthermore, a shorter period of time is necessary for visual deprivation to cause amblyopia than is the case for strabismic or anisometropic amblyopia. A18 It would be controversial to substitute surgery for glasses in a patient wit h fully accommodative esotropia. Surgery for the accommodative excess portion of the deviation at near to enable an older child to wear single-vision glasses or contact lenses is less controversial. Surgery for any nonaccommodativ e portion of an esodeviation, as long as it has been confirmed that the patient is wearing the full hyperopic cor rection, is commonly performed. A19 In the gradient method of determining the AC/A ratio, plus or minus lenses c an be used to evaluate the effect of accommodation on the deviation at a given distance. A20 In-office probing can be performed in children younger than 6 months but bec omes increasingly difficult as the child gets older. By 9-12 months of age, the child is usually too big to hold do wn and forcibly probe. A21 The success rate of probe and irrigation alone drops to 33% if not done by 2 years of age. A22 Obstruction of drainage below the lacrimal sac occurs in about 5% of full-te rm newborns. Usually, a thin mucosal membrane at the lower end of the NLD is the cause. Symptoms become manif est by age 1 month in 80%90% of cases. A23 The classic presentation for congenital glaucoma is epiphora, light sensitiv ity, and a hazy cornea. IOP should be measured at the beginning of the examination under anesthesia because anesthe sia has a rapid lowering effect on IOP. Refraction might show a myopic anisometropia, which helps confirm the diagnosis and may require treatment to prevent amblyopia. A24 Bilateral congenital cataracts are either inherited (due to a metabolic or i nfectious cause) or developmental. The genetics of cataracts are not well-known, and although cataracts can be asso ciated with a chromosomal defect, this is rare and analysis is usually not indicated. The one treatable metabolic disorder is galactosemia, which can be detected by looking at the red blood cell enzymes.

A25 Wyburn-Mason and Sturge-Weber syndromes are the only two phakomatoses that h ave no genetic inheritance pattern. Tuberous sclerosis, von Hippel-Lindau disease, and neurofibromatosis ar e typically autosomal dominant. Ataxia-telangiectasia has autosomal recessive transmission, and incontinentia pi gmenti is X-linked dominant. A26 Retinal angiomas. a. Retinal angiomas are often located in the peripheral fundus and may be asympt omatic when small. b. As the lesion grows, its capillaries may become more incompetent and allow fo r transudation of fluid. c. If the retinal edema and exudates are extensive enough to involve the macula, vision becomes compromised. d. The peak incidence of retinal angioma occurs a decade before its cerebellar c ounterpart. e. The tumors are bilateral in up to 50% of cases. f. Treatment with cryotherapy or laser photocoagulation may be especially effica cious with smaller lesions. A27 Brachycephaly BrachycephalyPremature bilateral coronal suture closure, a characteristic featur e of Crouzon syndrome. denotes the asymmetry caused by premature closure of a unilateral coronal suture . PlagiocephalyScaphocephaly PlagiocephalyScaphocephalyis the abnormal skull shape caused by premature sagitt al suture closure. A28 Undercorrections are much more common than overcorrections. Overcorrections can occur after suture adjustment. Slippage of a resected muscle causes an undercorrection. A29 Retinal detachments (especially in children) and endophthalmitis are rare ev en in cases with known scleral perforation. Although the incidence was as high as 10% in the 1970s, spatulated needles have made the complication much less common recently. Most surgeons recommend cryopexy or lase r therapy, although some do not. A30 Localized conjunctival injection and chemosis noted several days postoperati vely at the site of eye muscle surgery may be caused by a. Suture allergy. b. Poor closure of the conjunctival wound. c. Conjunctival inclusion cyst. A31 Transposition procedures and other surgeries involving three or more rectus muscles pose a special risk of anterior segment ischemia. Such ischemia can occur even following two-muscle sur gery, although typically in elderly patients with poor circulation or blood dyscrasias. Iritis and sector iris atrop hy are characteristic of anterior segment ischemia. A32 Botulinum toxin is particularly useful for small residual angles following s trabismus surgery. A33 Hospitalization, patching, bed rest, cycloplegia, and steroids are all contr oversial in the treatment of hyphemas, which should be individualized according to the risk of complications. Hyphemas are not more common in sickle cell disease or trait, but they are more dangerous because of impeded resorption and risk of retinal vascular occlusion. Because of the risk of corneal blood staining and the difficulty obtaining accur ate pressure measurements, total

hyphemas in young children should be evacuated in this time frame. A34 The cornea flattens during the first year of life, resulting in a decrease i n corneal power. A35 Changes of the eye parameters. a. The horizontal palpebral fissure at birth is almost as long as in childhood, yet a newborn's vertical fissure is only half the size of an adult's. b. The diameter of the eye at birth is about 66% of that in adulthood. The eye g rows rapidly during the first 2 years of life; growth then slows until puberty. c. Infants often have astigmatism during their first months of life, and most yo ung children are hyperopic. Absence of hyperopia during early childhood is usually a harbinger of myopia, wh ich increases as growth progresses. d. As the axial length of the eye increases (4 mm during the first 6 month of li fe), the lens flattens. At birth, the anteroposterior length of the eye is about 17 mm, increasing to approximately 24 mm in adulthood. e. The anterior chamber depth at birth is 2.3-2.7 mm, which is shallower than in adults because of the steep anterior lens surface in children. f. The average corneal horizontal diameter is 9.5-10.5 mm in newborns and 12 mm in adults. g. The radius of corneal curvature is 6.6-7.4 mm in newborns and 7.4-8.4 mm in a dults. Keratometry values change markedly in the first year of life, starting at approximately 52 D at bir th and flattening to 42-44 D in adulthood. Thereafter, keratometry remains relatively stable. h. The infant sclera is about half of its adult thickness and strength. A36 In Duane syndrome exotropia in gaze away from the affected eye of a unilater al case can sometime be seen. This finding is a feature differentiating type 1 Duane syndrome from sixth cranial nerve palsy. A37 Many cases of congenital motor nystagmus have a sensory visual defect. A38 Acquired unilateral cataracts in full size eyes are due to almost exclusivel y to posterior lenticonus or trauma. Both PFV and congenital nuclear cataracts are associated with microcornea. Lamel lar (zonal) cataracts are always bilateral. A39 In general posterior lenticonus behaves as an acquired cataract, and therefo re the visual potential is often good.

A40 Following strabismus surgery, Diplopia is more common with overcorrections. Perforation of the sclera usually causes a chorioretinal scar with no visual sequela. A41 IOL implantation in children with JIA remains controversial. A42 A cranial venous thrombosis can mimic an orbital pseudotumour. A43 The craniosynostoses are often associated with excyclorotation of the orbits and abnormal insertions of the horizontal rectus muscles. This may result in a V-pattern exotropia, in which th e exotropia increases in upgaze and is minimized in downgaze. It is also associated with secondary apparent IOOA. A44 The main difference between the phenotypes of Crouzon vs other craniofacial syndromes especially Apert is that Crouzon don t have hand or foot abnormalities. A45 All of the following are characteristic of the shaken baby syndrome. a. The child is younger than 12 months. b. Parenchymal brain damage and intracranial hemorrhage are common. c. Retinal hemorrhage is typical, especially in the posterior pole. d. Adnexae and anterior segments are typically uninvolved. C. . ORBIT, EYELID AND LACRIMAL SYSTEM ORBIT, EYELID AND LACRIMAL SYSTEM A1 The optic canal is located immediately superior and lateral to the sphenoid s inus wall. A2 The five major branches of the facial nerve include the temporal, buccal, mar ginal mandibular, cervical, and Zygomatic. A3 AAO neuro-ophthalmology. The Extracranial trunk of the nerve passes between t he superficial and deep lobes of the parotid gland, where it divides into two trunks, the temporofacial superiorly, a nd the smaller cervicofacial inferiorly. These further variably divide into five major branches: the temporal, zygomatic, infraorbital, buccal, and mandibular. The temporal and zygomatic branches laterally innervate the orbicularis oculi mu scles. The infraorbital and buccal branches may also variably contribute to the inferior orbicularis. A4 The parotidomasseteric fascia is generally a thin, wispy structure that overl ies the facial nerve branches, which overlie the masseter muscle, which is therefore the deepest of the structures li sted. A5 A 65-year-old woman presents with a progressively enlarging mass in the right inferior orbit. Distraction of the lower eyelid reveals a "salmon patch" appearance to the fornix. Approximately 90% of o rbital lymphoproliferations will prove monoclonal, suggesting a diagnosis of lymphoma; while 10% are polyclonal, suggesting reactive lymphoid hyperplasia. A6 Eyelid retraction is the most common clinical feature of Graves's ophthalmopa thy (and Graves's ophthalmopathy is the most common cause of eyelid retraction). A7 Orbital abscesses in adults are more likely to arise from chronic sinusitis. These infections are more likely to be mixed polymicrobial infections of aerobic and anaerobic bacteria. A8 Subperiosteal abscess of the orbit in children is more likely than in adult t o respond to single antibiotic therapy. In patient younger than 9 years, most of these infections are due to single organis ms affecting the ethmoid sinuses, and they may drain spontaneously if vision is not threatened. A9 Although Graves ophthalmopathy occurs most commonly in association with hyper thyroidism (and treatment of hyperthyroidism is important in the overall care of the Graves patient), the cou

rse of the ophthalmopathy does not necessarily parallel the activity of the thyroid gland or the treatment of thyro id abnormalities. A10 The most common location for orbital lymphoma is lacrimal fossa. Up to 50% o f orbital lymphoproliferative lesions arise in the lacrimal fossa. A11 Although systemic steroids are useful in idiopathic orbital inflammatory syn drome (orbital pseudotumor), they are not recommended in the treatment of lymphoproliferative lesions. Radiotherap y is the treatment of choice for patients with localized ocular lymphoproliferative disease. A surgical cure usua lly cannot be attained because of the infiltrative nature of lymphoid tumors. Patients must be monitored indefinitely for development of additional lymphoproliferative lesions. A12 Treatment recommendations vary for isolated MALT lesions. Simple excision ma y be diagnostic and therapeutic, with some authors recommending observation for isolated stable lesi ons. A13 Surgery should be avoided for patient with orbital varices except when pain or visual loss necessitates treatment. A Valsalva maneuver during CT scan may be required for diagnosis. Eno phthalmos on the involved side is common. A14 At 4 days after injury with exophthalmos and restricted motility, additional observation is necessary to allow traumatic edema to subside. This subsidence may be enhanced by the administratio n of oral prednisone, 1 mg/kg per day for 1 week. Urgent exploration of orbital blowout fractures is necessary only if there is radiographic evidence of gross extraocular muscle entrapment beneath the fracture fragments. It is gen erally preferable to allow 10-14 days for swelling to resolve and motility to be reevaluated before proceeding with su rgical repair. A15 the following features is most likely to be found on an orbital CT scan of a patient with Graves ophthalmopathy is an increased amount of orbital fat in the presence of normal-sized extraocula r muscles. A16 Capillary hemangioma is the most common benign primary orbital tumor among c hildren. Orbital cellulitis is the most common cause of unilateral exophthalmos in children. Metastatic neuroblasto ma is the most common

metastatic cancer of the orbit in children. Neurofibromas are rarely malignant a nd are uncommon orbital tumors in children. Graves's ophthalmopathy is very rare in children. A17 Basal cell carcinomas occur approximately 40 times more often than either se baceous carcinomas or squamous cell carcinomas. Even though both sebaceous cell carcinomas and squamou s cell carcinomas occur more often in the upper eyelid than they do in the lower and basal cell carcinom a occurs more often in the lower lid than in the upper, the far greater frequency of basal cell carcinoma results in its still being the most common malignant neoplasm of the upper eyelid. (The tumour has a predilection for the l ower eyelid and the lid margin. Kanski). A18 The majority of histologically malignant lymphoid lesions of the orbit are r elatively indolent or low-grade lymphomas. The majority of orbital lymphomas are well differentiated. A19 Sudden pain associated with tense, ecchymotic eyelids following blepharoplas ty is indicative of a postoperative orbital hematoma. If there is no decreased vision, altered pupillary response, o r other indication of decreased optic nerve function, the patient may be managed conservatively with close observation . If there is any evidence of optic nerve or ocular compromise, the wounds should be opened immediately and drains i nserted to decompress a possible orbital hemorrhage. A lateral canthotomy and inferior cantholysis shoul d be performed next if the above is not effective. A20 Posttraumatic ptosis, immediate treatment. If ptosis is present, the wound s hould be explored. If orbital fat (preaponeurotic fat pad) is visible, exploration of the deeper orbital structure s should be undertaken. Repair of the levator aponeurosis is indicated as a primary procedure. A21 Medial canthal trauma usually results from tangential (lateral) forceful tra ction on the lower eyelid causing an avulsion type of injury at the weakest point of the eyelid, the medial canthal t endon. This frequently involves the canalicular system. Repair can be easily delayed 12-24 hours. Delayed lacrimal r epair (>6 months) will require a CJDCR with Jones tube. A22 Indications for repair of orbital blowout fracture include all of the follow ing: a. Cosmetically unacceptable enophthalmos (2 mm or more of enophthalmos). b. Fractures involving more than half of the orbital floor. c. Pain and oculocardiac reflex on upgaze. d. Significant inferior rectus entrapment. A23 Early implant extrusion is typically associated with either poor wound closu re or an implant that is too large. Early implant extrusion is typically associated with either poor wound closure o r an implant that is too large. Late implant extrusion can be associated with tumor recurrence. Late implant ext rusion can be associated with a Late implant extrusion can be associated with tumor recurrence. Late implant ext rusion can be associated with a . conjunctival cyst conjunctival cyst A24 The most important step in repair of a full thickness eyelid laceration is c areful approximation of the tarsus. The tarsus is the backbone of the eyelid and must be appropriately approximated so t

hat the repair is stable. A25 The most important determinant in selecting a corrective procedure for any t ype of ptosis is amount of levator function. A26 Invasive aspergillus infections of the orbit. a. In addition to acute fulminant fungal sinusitis with orbital invasion, asperg illosis can cause chronic indolent infection resulting in slow destruction of the sinuses and adjacent structures ( bone). b. Corticosteroids may produce an initial clinical improvement. c. Septate branching hyphae of uniform width are seen histologically. D. . EXTERNAL DISEASE AND CORNEA EXTERNAL DISEASE AND CORNEA A1 In general, visual acuity is reduced earlier when there is corneal surface ir regularity as opposed to diffuse clouding with a smooth epithelial surface. Therefore, dystrophies such as primary familia l amyloidosis, where amyloid occurs as subepithelial deposits; Reis-Bucklers dystrophy, which affects Bowman's layer ; and lattice dystrophy, where amyloid is deposited most heavily in the anterior stroma and subepithelial area, would all cause some vision difficulty by the second decade of life. Central cloudy dystrophy of Francois, on the other hand, appears as deep, polygonal, gray areas with an unaffected anterior stroma and smooth epithelium. Vision is t ypically not reduced by this condition. A2 If corneal surface irregularity from severe anterior basement membrane dystro phy is the main cause of decreased visual acuity, overrefraction with a rigid gas-permeable contact lens will mask the surface irregularity and improve visual acuity, so we can differentiate that VA is reduced due to anterior surfac e irregularity not due to cataract. Glare testing is of no benefit since both corneal pathology and cataract can manifest decreased vision with glare testing. A3 Although fluorescein has no antiviral properties, rose-bengal, a halide deriv ative of fluorescein, has antiviral and other toxic effects. A4 Although the mucin layer is the innermost of the three layers and is secreted by conjunctival goblet cells, it is a hydrophilic layer that overlies the hydrophobic ocular surface. A5 In lacrimal gland dysfunction states, the normal production of proteins by th e lacrimal gland is diminished. A decreased tear lysozyme or lactoferrin level is highly suggestive of dry eye. Te ar film osmolarity has been shown to be increased in patients with aqueous tear deficiency as well as those with m eibomian gland dysfunction. The tear meniscus is normally 1.0 mm in height and convex. A tear meniscus 0.3 m m or less is considered abnormal.

A6 Sarcoidosis is a multisystem disorder of uncertain etiology. After pulmonary involvement, ocular involvement is the most common manifestation, seen in up to 50% of affected patients. The most comm on corneal manifestation is calcific band keratopathy, often associated with chronic uveitis or elevated ser um calcium levels. Nummular keratitis, Thickening of Descemet's membrane, and deep stromal vascularization can occur wi th sarcoidosis. A7 The application of dilute (1%) bleach to tonometer tips for at least 10 minut es is recommended to prevent transmission of adenoviruses. A8 The Acanthamoeba life cycle includes the motile trophozoite and the dormant c yst forms. Free-living amoebae are found in both forms, but only the trophozoite is infectious. Both forms are foun d in infected human tissues. Acanthamoeba can be grown on blood agar and on buffered charcoal yeast extract a gar. Acanthamoeba can be visualized with calcofluor white, acridine orange, or the Giemsa stain. Acantham oeba keratitis can appear as a pseudodendrite early in its course. A9 EKC is the only adenoviral syndrome with significant corneal involvement. The infection is bilateral in a majority of patients and may be preceded by an upper respiratory infection. Photophobia and reduced vision from adenoviral subepithelial infiltrates may persist for months to years. It is not associated with enlarged corneal nerves, which are seen in such conditions as MEN syndrome and leprosy. It may be associated with l arge central geographic corneal erosions. A10 Herpes zoster ophthalmicus (HZO). The current recommendation for HZO is oral famciclovir 500 mg three times per day, valacyclovir 1000 mg three times per day, or acyclovir 800 mg fiv e times per day for 7-10 days, best if started within 72 hours of the onset of skin lesions. Topical antiviral medicati ons are not effective. Intravenous acyclovir therapy is indicated in patients at risk for disseminated zoster due t o immunosuppression. A11 Graft-versus-host disease Graft-versus-host disease . . a. A relatively common complication of allogenic bone marrow transplantation per formed most commonly for hematopoietic malignancies. b. Conjunctival inflammation in GVHD can be severe and even associated with limb al stem-cell deficiency and secondary corneal scarring, but the latter is a rare complication. c. Typical presentations include conjunctival and corneal epithelial erosions, a queous tear deficiency, and cicatricial lagophthalmos. d. Aggressive lubrication and punctal occlusion are the mainstays of local thera py. Severe filamentary keratopathy can be additionally treated with mucolytic agents (10% acetylcystein e) as needed. Severe ocular surface disease in GVHD may be associated with active nonocular (often sk in) GVHD and may require increased systemic immunosuppression by cyclosporine or tacrolimus (FK50 6). A12 Although congenital syphilis was the first infection to be linked with inter stitial keratitis, many other agents can

also cause IK as (Herpes simplex virus, Herpes zoster virus, and Chlamydia trach omatis). A13 Several disorders can be associated with limbal stem-cell dysfunction, inclu ding aniridia, contact lens-induced keratopathy, atopic keratoconjunctivitis, ocular cicatricial pemphigoid, Stevens -Johnson syndrome, chemical injuries, and keratitis associated with multiple endocrine neoplasia (MEN IIb or III). A14 Surgical treatment of squamous cell carcinoma should include all of the foll owing: a. Excision of conjunctiva 4 mm beyond the clinically apparent margins of the tu mor. b. Resection of a thin lamellar scleral flap beneath the tumor. c. Treatment of the remaining sclera with absolute alcohol. d. Cryotherapy applied to the conjunctival margins. A15 The combined ocular and cutaneous pigmentations seen in patient are referred to as oculodermal melanocytosis. a. Approximately 5% of cases are bilateral. b. Malignant transformation is possible but rare and seems to occur only in pati ents with a fair complexion. c. Malignant melanoma can develop in the skin, conjunctiva, uvea, or orbit. d. The lifetime risk of uveal melanoma in a patient with ocular melanocytosis is about 1 in 400, significantly greater than the risk of 1 in 13,000 of the general population. A16 Malignant transformation of primary acquired melanosis should be suspected w hen a lesion shows nodularity, enlargement, or increased vascularity. Small typical areas of primary acquired m elanosis can be observed and biopsy delayed until progression is observed. A17 Corneal clouding at birth has many different etiologies, including congenita l glaucoma, birth trauma, sclerocornea, Peter's anomaly, or CHED2. A18 Two forms of CHED are recognized. The more common autosomal recessive type ( CHED2) presents at birth, remains stationary, and is accompanied by nystagmus. The bluish white cornea may be two to three times' normal thickness and have a ground-glass appearance, but this finding is not associated with tearing or photophobia. The dominant form (CHED1) presents in the first or second year of life, although exp ressivity is variable. It is slowly progressive and accompanied by pain, photophobia, and tearing, but nystagmus is not present. The cornea exhibits a diffuse blue gray ground-glass appearance. A19 The most distinctive microscopic finding in posterior polymorphous dystrophy is the appearance of abnormal, multilayered endothelial cells that look and behave like epithelial cells or fib roblasts. These cells

a. Show microvilli. b. Stain positive for keratin. c. Show rapid and easy growth in cell culture. d. Have intercellular desmosomes. e. Manifest proliferative tendencies. f. Both angle closure and open-angle glaucoma can occur, and 14% of patients hav e elevated IOP. A20 The frequency of regraft is highest in lattice dystrophy, followed by granul ar dystrophy and then macular dystrophy. Fuchs dystrophy does not typically recur in the graft. LGMF. A21 Schnyder crystalline corneal dystrophy: features of this rare, slowly progre ssive autosomal dystrophy include central subepithelial crystals (seen only in 50% of patients), central corneal o pacification, dense corneal arcus lipoides, mid peripheral corneal opacification, and decreased corneal sensation. A22 Calcific band keratopathy: this calcific degeneration of the superficial cor nea involves mainly Bowman's layer. There are six main causes: chronic ocular disease; hypercalcemia caused by hyper parathyroidism, vitamin D toxicity, milk-alkali syndrome, sarcoidosis, and other systemic disorders; hered itary transmission; elevated serum phosphorus with normal serum calcium; chronic exposure to mercurial vapors or to mercurial preservatives in ophthalmic medications; and silicone oil instillation in an aphakic eye. A23 An unusual syndrome, diffuse lamellar keratitis (DLK, sands of the Sahara, S OS) with diffuse interface haze, may appear on postoperative day 1-3 with possible decrease in vision. The condit ion appears to be a nonspecific inflammatory reaction in response to a variety of toxic insults. Central or peri pheral WBCs with granular appearance usually respond to frequent topical steroids; however, when dense clumps appear in the visual axis, lifting the flap and irrigating, followed by intensive topical steroid treatment is usually requi red. For severe cases, systemic steroids may be used adjunctively. A24 Bilateral herpes simplex occurs in about 3% of the cases and is unrelated to treatment or other herpetic viral infections. It may occur with HIV infections, but ocular atopy is much more freq uent. A25 Eyes with posterior choroidal melanoma may be considered acceptable; however , most medical eye bank directors decline their usage. All other conditions exclude the donor cornea (Le ukemia, Previous PRK for correction of 2 D of myopia, Active septicemia). A26 Dry eye can be detected with corneal topography which shows as irregular ast igmatism from disruption of the tear film. A27 A small wound leak (with no anterior chamber shallowing) seen on the first p ostoperative visit after penetrating keratoplasty. The wound is always checked carefully for leakage on postoperative visits. Small wound leaks that do not cause anterior chamber shallowing frequently close spontaneously. Patching, therapeutic contact lenses, and use of inhibitors of aqueous production may hasten wound closure. Resuturing is advised for leaks lasting longer than 3 days. A28 Corneal transplant rejection takes three clinical forms-epithelial, subepith elial, and stromal-which may occur

either singly or in combination. In epithelial rejection, lymphocytes cause an e levated, linear epithelial ridge that advances centripetally. In subepithelial rejection, subepithelial infiltrates ar e seen, sometimes accompanied by a cellular anterior chamber reaction. Isolated stromal rejection is not common but can be seen as stromal infiltrates and neovascularization. In very aggressive severe or prolonged bouts of graft re jection, the stroma can become necrotic. Endothelial rejection is the most common and serious form of corneal t ransplant rejection. Inflammatory precipitates are seen on the endothelial surface in fine precipitates, in random clumps, or in linear form (Khodadoust line). Inflammatory cells are usually seen in the anterior chamber as well. E. . INTRAOCULAR INFLAMMATION AND UVEITIS INTRAOCULAR INFLAMMATION AND UVEITIS A1 Significant inflammation is usually seen in sympathetic ophthalmia and phacoa ntigenic endophthalmitis. Milder inflammation is usually seen in Propionibacterium acnes endophthalmitis. In phac olytic glaucoma, macrophages engulf leaking lens protein and there is usually little or no intraocular inflam mation. A2 Patients with rheumatoid arthritis, Behçet syndrome, and Reiter syndrome are mo re likely to have a nongranulomatous uveitis and patients with sarcoidosis, a granulomatous uveitis. A3 Prednisolone acetate 1% (Pred Forte) and prednisolone phosphate 1% (Inflammas e Forte) are more effective than loteprednol (Lotemax), fluorometholone 0.25% (FML Forte), and dexamethasone 0.1% (Decadron) in treating intraocular inflammation in uveitis. A4 Kawasaki syndrome is usually seen in childhood. 85% of patients are less than 5 years old. A5 Rigid, closed-loop anterior chamber intraocular lenses are most associated wi th recurrent uveitis. Iris plane intraocular lenses occasionally cause inflammation, and, less commonly, a sulcus -placed posterior chamber intraocular lens causes inflammation from iris chafing or uveitis-glaucoma-hyphe ma (UGH) syndrome. Silicone intraocular lenses have been associated with iritis in the past. A6 In one series of JIA-associated uveitis cases, cataract was seen in 84%, band keratopathy in 70%, macular edema in 42%, and glaucoma in 26% of patients. A7 Periocular corticosteroid injections may cause proliferation of the organism (toxoplasmosis) and should be avoided. A8 The IgM antibody titer is the best laboratory test for a newly acquired toxop lasmosis infection. The IgM antibody titer will be elevated early after the infection but will not be detectable 2-6 months after the initial infection.

A9 Ophthalmologic signs of systemic lupus erythematosus include cotton-wool spot s, retinal arteriolitis and vascular occlusion, vitreous hemorrhage, choroidal infarction, choroiditis, and subretina l exudation. Anterior uveitis is not usually seen with systemic lupus erythematosus. A10 A salt-and-pepper fundus may be seen in congenital syphilis, and uveitis may be seen in secondary syphilis and in other stages. A lumbar puncture should be performed in patients with uvei tis and syphilis. Syphilic uveitis can be cured with proper treatment, even in patients with AIDS. A11 In tertiary syphilis, serum FTA-ABS and MHA-TP, and CSF VDRL are positive. H owever, serum VDRL may be negative in a patient with tertiary syphilis. A12 Blunt trauma is not likely to incite sympathetic ophthalmia. A13 In an older patient with a nonresponsive uveitis, vitritis, subretinal infil trates, and neurological signs, a largecell, non-Hodgkin lymphoma should be suspected. A CT scan or MRI of the head wou ld be the first important step in this patient's management. CSF fluid analysis or vitreous biopsy can confirm the diagnosis. Sarcoidosis, tuberculosis, or giant cell arteritis are possible; however, it is important to rule out the possibility of a large-cell lymphoma. A14 The Bacillus cereus organism is frequently (26%-46%) found in traumatic endo phthalmitis and may cause a fulminant endophthalmitis. A15 Bleb-related endophthalmitis may present months or years after glaucoma filt ering surgery. The visual prognosis is poor. Haemophilus influenzae and a Streptococcus species are common causative agents of blebrelated endophthalmitis. In eyes with endophthalmitis after cataract surgery or secondary intraocular lens implantation, a preoperative visual acuity of better than light perception argue s for a "tap and inject" rather than vitrectomy Endophthalmitis Vitrectomy Study). However, the EVS conclusion is not applicable to patients with blebrelated endophthalmitis, and a vitrectomy is usually necessary in severe bleb-re lated endophthalmitis. A16 In one study, perinatal transmission was the least frequent (1%) mode of tra nsmission of HIV. Sexual intercourse (70), intravenous drug abuse (27), and blood transfusions (2%-3%) we re more common causes of HIV transmission. A17 Anterior uveitis is the most common morphologic form of uveitis worldwide. M en and women appear to be equally affected. The paediatric age group has the lowest incidence and prevalen ce and the age over 65 years of age has the highest prevalence and incidence. A18 Cataract may occur after intravitreal injection in phakic patient, especiall y if multiple injections are given. Ocular hypertension may occur in 25% of patients requiring topical therapy, but up to 10% (same book 1%) may need surgical treatment of their glaucoma. Patients with CME may benefit from in travitreal corticosteroid injection, but the effect rarely last more than 6 months. A19 Yanoff. Intravitreal triamcinolone. Approximately a quarter of patients had a rise in intraocular pressure requiring therapy, including 1% of patients who required glaucoma surgery. Patie nts receiving intravitreal

triamcinolone have approximately a 40% risk of developing increased intraocular pressure and 100% chance of developing a cataract. A20 Late-onset, pauciarticular, JRA/JIA-associated iridocyclitis is usually acut e, recurrent, and symptomatic. It has somewhat better long term visual prognosis than ANA-positive, early onset pa uciarticular group. 75% of patients are HLA-B27 positive. Many go on to develop seronegative spondyloarthropathy lat er in life. Older boys are most often affected. A21 Behçet, necrotizing scleritis, SO, chronic recurrent VKH are considered absolu te indications for early institution of immunomodulatory therapy. A22 APMPPE may be associated with cerebral angiitis which may be fatal if untrea ted with IV corticosteroids. A23 A patient with bilateral granulomatous panuveitis whose workup revealed a po sitive PPD skin test would require treatment with the following medications, isoniazid, rifampicin, pyrazin amide, and corticosteroid. A24 Sarcoidosis, MS, birdshot RC, and Eales disease are associated with retinal vasculitis affecting venules. SLE, PAN, and necrotizing herpetic retinitis are associated with retinal vasculitis p rimarily affecting arterioles. A25 The most common cause of hypopyon AU in USA is HLA-B27 associated disease. I diopathic AU will not typically result in hypopyon. A26 Fuchs heterochromic iridocyclitis. It is chronic and associated with catarac t in 50% and glaucoma in 60% of patients. A27 Yanoff. Cataract is seen in over 80% and elevated IOP in 25 60% of patients, a rising either from the inflammation or from corticosteroid therapy. Posterior subcapsular cataracts are most common, and patients tend to tolerate cataract extraction without much difficulty. Glaucoma develops in 6.3 59% of patients, and 25 60% of those individuals require a filtering procedure during the course of treatment. Most i mportantly, the complications from chronic inflammation in FHI do not usually include posterior synechiae or cystoi d macular edema, unlike other anterior uveitis entities. Fine, stellate keratic precipitates can be seen scatt ered over the entire endothelial surface; but all types of precipitates, including the granulomatous mutton-fat type, have b een described. Bilateral disease may be discerned by subtle signs, such as loss of detail on the iris surface, an d by the more specific sign of diffuse stellate keratic precipitates A28 p-ANCA is associated with PAN, relapsing poly chondritis and renal vasculiti s.

A29 MEWDS. Recurrences are uncommon, occurring in 10-15% of patients, and bilate ral disease is rare. The prognosis is excellent, with vision recovering completely in 2-10 weeks without treatment. A30 Granular vs fulminant appearance in CMV retinitis is a function of retinal t hickness. Peripheral CMV retinitis appears granular. A31 Extremely low CD4 counts, fewer than 50 cells/mm3, are associated with CMV r etinitis. A32 A PPD skin test is imperative prior to therapy with infliximab because milit ary or disseminated TB may occur in patient treated with infliximab who have latent undetected TB. F. . GLAUCOMA GLAUCOMA A1 Average central corneal thickness ranges between 536 and 544 .m, depending on the type of pachymeter used.

A2 In AGIS, the Advanced Glaucoma Intervention Study, patients had significantly better outcomes if their IOP was controlled below 18 mm Hg at all visits. A3 In the CNTG, Collaborative Normal-Tension Glaucoma Treatment Study, progressi on was reduced by nearly threefold by a reduction in IOP of 30% (The target pressure set in CNTG was 30%) . The Collaborative NormalTension Glaucoma Study (CNTGS) found that reducing IOP by greater than 30% reduc ed the rate of visual field progression from 35% to 12%, confirming a clear role of IOP in this disease. A4 In the GLT, the Glaucoma Laser Trial, the group treated with laser, compared to the group treated with medical therapy, showed nearly equivalent outcomes (The GLT found that both arms of the study did equally well). A5 A feature associated with exfoliation syndrome are: a. Spontaneous lens dislocation. b. Earlier cataract formation. c. Higher incidence of vitreous loss during cataract surgery. d. Volatile IOPs. e. Narrow anterior chamber angles. A6 In pigmentary dispersion syndrome with elevated IOP. a. Patients with pigmentary dispersion syndrome are usually myopic with increase d pigmentation of the trabecular meshwork. As a result, they require less energy with laser trabeculop lasty and have a higher incidence of hypotony maculopathy. b. Laser peripheral iridectomy may flatten the peripheral iris contour but will not deepen the anterior chamber in this condition. A7 Pooled data from large epidemiologic studies indicate that the mean IOP is ap proximately 16 mm Hg, with a standard deviation of 3 mm Hg. IOP, however, has a non-gaussian distribution wit h a skew toward higher pressures, especially in individuals over age 40. For the population as a whole, no clear l ine exists between safe and unsafe IOP. Screening for glaucoma based solely on IOP >21 mm Hg may miss up to half of the people with glaucoma in the screened population. A8 Medications have limited long-term value for congenital and infantile glaucom

a in most cases, and the preferred therapy is surgical. The initial procedures of choice are goniotomy or trabeculo tomy if the cornea is clear, and trabeculotomy ab externo if the cornea is hazy. Brimonidine should not be used i n infants and topical beta blockers should be used cautiously. A9 Use of brimonidine in infants and young children should be avoided due to an increased risk of somnolence, hypotension, seizures, apnea, and serious derangements of neurotransmitters in t he CNS, presumably due to increased CNS penetration of the drug. A10 Automated perimetry uses a variety of "staircase" strategies to estimate the threshold sensitivity. The strategy chosen will affect the speed and reproducibility of the visual field produced. A11 The prevalence of glaucoma in the black population is estimated to be 3 to 6 times higher than in the white population. A12 The 6 described POAG loci all appear to be inherited in an autosomal dominan t pattern. A13 It is surprising that the 3 genes identified for primary congenital glaucoma are estimated to account for 75% of all known forms of the disease. A14 The data are the least compelling for diabetes being a risk factor for highpressure POAG. A15 Yanoff. Population-based studies fail to consistently show an association be tween diabetes and POAG, although two studies on Hispanics and Whites in the United States showed a weak association between the two conditions A16 Although initial success rates are high after laser trabeculoplasty, the suc cess rate declines over time to approximately 50% after 3-5 years and 30% after 10 years. A17 Many patients require more than one glaucoma medication to control their dis ease. Adverse reactions may occur with any glaucoma medication. These reactions resolve when the medication is discontinued. Usually an alternative medication or laser trabeculoplasty can be used to successfully trea t the patient. A18 Use of mitomycin C during filtration surgery has been associated with persis tent ocular hypotony, bleb leaks, and infections. The blebs are often less vascular than the surrounding tissues t hat are not treated with mitomycin C.

A19 The usual primary glaucoma surgery is trabeculectomy. If patients have faile d prior trabeculectomy, have inadequate conjunctiva (eg, due to extensive prior ocular surgery), or have a po or prognosis for successful trabeculectomy (eg, active uveitis, neovascular glaucoma, ICE syndrome), they ma y be candidates for drainage implant surgery. A20 Cyclophotocoagulation may be associated with visual loss, hypotony, pain, in flammation, cystoid macular edema, hemorrhage, and even phthisis bulbi. Sympathetic ophthalmia is a rare but serious complication. A21 The IOP in eyes with low scleral rigidity may be underestimated with applana tion tonometry, although this effect is more pronounced when techniques of indentation tonometry are used. A22 ICE syndrome has an abnormal corneal endothelium that allows for the PAS to extend anterior to Schwalbe's line. Neovascular glaucoma and Fuchs heterochromic iridocyclitis have a normal c orneal endothelium. In AxenfeldRieger syndrome, Schwalbe's line is displaced anteriorly; however, the PAS are l imited to this anterior displacement. A23 Elevated episcleral venous pressure may be associated with: a. Sturge- Weber syndrome. b. Facial cutaneous angiomas, such as nevus flammeus. c. Thyroid ophthalmopathy. d. Dilation of episcleral vessels. e. Foreshortening of the conjunctival fornices not associated with elevated EVP. A24 The 6 described POAG loci all appear to be inherited in an autosomal dominan t pattern. A25 It is surprising that the 3 genes identified for primary congenital glaucoma are estimated to account for 75% of all known forms of the disease. A26 OHTS. Although all patients who were treated initially receive 1 medication and the target IOP lowering 20%, approximately 50% require at least 2 medications to maintain the target at 5 yea rs. The average IOP lowering was 22.5% and the treated group had significantly lower conversion rates to glaucoma . G. . LENS LENS A1 Major epidemiologic studies confirm an increased prevalence of cataract in wo men. A2 Current smoking, white race, and lower education are all risk factors for nuc lear opacification. A3 A small capsulorrhexis leaves more anterior capsule, which leads to greater r esistance in nuclear rotation for quadrant removal techniques, as well as increased difficulty in chopping techniq ues. The zonular laxity allows the larger anterior capsule remaining to contract to a much smaller opening. YAG ant erior capsular relaxing incisions can be made in the early postoperative period to reduce the anterior capsular ph imosis, which can further reduce the zonular integrity. A4 With mature lenses, laser interferometry is incapable of passing through the lens to allow accurate responses. However, patients with mature lenses and normal macular function may be able to discriminate entoptic phenomena, color, and even the placement of 2 lights separated from one another. A5 The lens may be dislocated into the posterior chamber without loss of visual

function. Couching of the lens was an accepted method of restoring vision before lens removal was attempted first by D aviel in the 17th century. If the posterior dislocation of the lens is accompanied by inflammation or disruption o f the lens capsule, removal may be indicated by a pars plana route. A6 Recent studies have shown a higher incidence of endophthalmitis with clear co rneal incisional though the mechanism has not been demonstrated. Both poorly constructed wounds and wound bu rns can lead to a leaky wound, which many suspect may be the cause of increased infection rates. Recent experimental studies have demonstrated that transient reduction in postoperative IOP may also result in po or wound apposition in clear corneal incisions, with potential for fluid flow across the cornea and into the anterior chamber, with the attendant risk for endophthalmitis. A7 Postoperative ptosis don t eliminated with topical anesthesia because still may occur due to trauma from lid speculum or postoperative ocular inflammation which weaken levator muscle. A8 Loss of vitreous is not a problem for the eye; vitreous traction is. When rem oving vitreous, the goal is to prevent any possibility of traction by removing sufficient vitreous to keep it away from oth er intraocular structures such as the IOL or away from the wound. Therefore, vitrectomy is not complete until all vitreous is removed anterior to the posterior capsule, thus ensuring less risk of traction. This is the best way to decrease t he chances of postoperative ME. A9 Early detection of capsular rupture is critical to the satisfactory resolutio n of this unexpected occurrence. As soon as the surgeon notes a rupture or suspects one, the first thing to do is stop worki ng and freezes the action. This prevents further trauma to the capsule and allows for a calm assessment of the s ituation before proceeding any further. Without removing the phaco handpiece, viscoelastic can be instilled thr ough the paracentesis port to freeze the action; the phaco is then removed. The next step is determined by the extent of the rupture, how much nucleus is left to be removed, and the presence or absence of vitreous. A10 Early morning Pachymetry is better predictor for postoperative endothelial f unction than specular microscope. Since the endothelium is under greater stress in the early morning. Normal corneal pachymetry measurements obtained in the early morning suggest that the cornea will probably remain clear following cataract surgery.

A11 Occlusion of the phaco tip reduces or interrupts fluid evacuation through th e phaco handpiece. This results in an increased buildup of heat within the handpiece and a transfer of thermal ener gy to the incision. Use of lower ultrasound power reduces heat buildup. Aspiration of viscoelastic, use of more e asily aspirated cohesive viscoelastics, higher aspiration flow rates and vacuum levels, and a loose fit b etween the phaco handpiece and the incision all contribute to a more efficient fluid flow through the handpiece and /or the incision, reducing the transfer of thermal energy. A12 Posterior infusion syndrome causes shallowing of the anterior chamber during cataract surgery, not in the postoperative period. This rare complication typically occurs during hydrodissec tion, when fluid may be misdirected into the vitreous cavity, resulting in forward displacement of the lens. Wound l eakage and suprachoroidal hemorrhage may result in a flat chamber during or following surgery. Suprachoroi dal effusion, pupillary block, and ciliary block with aqueous misdirection occur in the postoperative period. Supra choroidal effusion is often associated with hypotony and may be associated with a wound leak. Pupillary and ciliary blo ck, as well as suprachoroidal hemorrhage, is often associated with normal or elevated IOP. A13 If the capsulorrhexis tear starts to extend too far peripherally, the flap c an sometimes be salvaged and the tear brought more centrally. First, the surgeon should check for positive vitreous pr essure associated with forward displacement of the lens. This may be caused by the capsulotomy instrument, the surgeon's fingers, or the lid speculum pressing against the globe; and it can be corrected. Refilling the ante rior chamber with viscoelastic, and/or inserting a second instrument (such as an iris spatula) through the paracentesis to press posteriorly on the lens may help reduce forward displacement of the lens and allow for redirection of the ca psular tear. Using the bent cystotome needle to redirect the tear centrally may also be helpful, as this instrument ca uses minimal wound distortion when inserted in the eye, and it can create sharp changes in the direction of the tea r over very short distances. Often, several of these just-described maneuvers need to be employed in order to redire ct and salvage the capsulorrhexis. A14 The surgeon must distinguish carefully between an actual cataract and appare nt lens opacity due to fibrin coating on the anterior lens capsule of an otherwise clear lens. Cataracts shoul d never be extracted through a corneal laceration; this procedure would cause additional injury to an already t raumatized corneal endothelium. Traumatic cataracts can develop long after the actual ocular injury. The use of phacoemulsification through a limbal incision would not be preferred when the view through the cornea is inadequate. In this situation, cataract surgery should either be postponed or, if necessary, combined with penetrating keratopla sty. When the lens is subluxated as a result of zonular dehiscence, a pars plana approach is preferred. Insertion of an IOL as part of a combined cataract extraction and corneal laceration repair procedure is controversial. Preoperativ e biometry is usually impossible and/or inaccurate, which can lead to significant anisometropia. Zonular status,

as well as capsular integrity, may not be certain. A15 Systemic .1A antagonists (for treatment of benign prostatic hypertrophy) may cause intraoperative floppy iris syndrome (IFIS) by competitively bind to the postsynaptic nerve endings of the i ris dilator muscle for long periods. Atrophy of the iris dilator results in intraoperative billowing of the iris and progressive miosis. H. . RETINA AND VITREOUS RETINA AND VITREOUS A1 The Central Vein Occlusion Study (CVOS) showed a benefit from panretinal phot ocoagulation when neovascularization occurred (not immediately) but no benefit from gird photocoag ulation for macular edema in older patients. In younger patients, there was a trend toward benefit from grid photoc oagulation, but this was not statistically significant. A2 The Central Vein Occlusion Study Group (CVOS). Thus, even though grid laser t reatment in the macula reduced angiographic evidence of macular edema, it yielded no benefit in improving visua l acuity. However, there was a trend in the CVOS that grid laser treatment might be beneficial in improving visual ac uity for eyes with macular edema in younger patients. A3 Although the Early Treatment Diabetic Retinopathy Study (ETDRS) reported that focal laser photocoagulation should be applied in patients with clinically significant macular edema even if the vis ion is 20/20, this was not mandated, and patients can be observed closely, especially if most of the edema is in the fove al avascular zone. Macular edema in the ETDRS was defined by clinical examination, not imaging methods like fluoresc ein angiography and optical coherence tomography (OCT). The DCCT findings, although correct, apply only to p atients with type I diabetes. The Diabetic Retinopathy Study (DRS) reported that immediate panretinal photocoagula tion should be applied with highrisk proliferative diabetic retinopathy. A4 The patient suffered a branch vein occlusion. The Branch Vein Occlusion Study (BVOS) reported that grid photocoagulation should be applied if macular edema is present for more than 3 m onths and no retinal hemorrhages would prevent laser treatment. PRP should be applied when neovascularization occ urs-not if ischemia (>5 disc diameters of capillary nonperfusion) is present. Because patients with macular n on-perfusion were excluded from the BVOS, the study findings do not apply. Embolic workups are not required in branc h vein occlusion. A5 The Studies of Ocular Complications of AIDS (SOCA) trials reported that patie nts treated with ganciclovir had a 79% higher mortality than those assigned to foscarnet, with both drugs equal in cont rolling CMV retinitis and with tolerable side effects. Retinitis progression was best controlled with combination treatme nt. Color photographs are the gold standard to follow for disease progression.

A6 Fundus albipunctatus is a form of congenital stationary night blindness chara cterized by striking yellow-white dots in the posterior pole. Patients have normal visual acuity and color vision. The rod ERG is minimal but normalizes after patients spend several hours in a dark environment. It is non-progressive and sh ould be differentiated from retinitis punctata albescens, which is a variant of retinitis pigmentosa. Nyctalopia is a feature of Fundus albipunctatus. A7 Reduction and delay of cone (or rod) b-waves signifies damage to cells diffus ely throughout the retina. This can occur in dystrophic disease, such as retinitis pigmentosa, in widespread ischemi c disorders such as central vein occlusion, and in diffuse infections or inflammations such as syphilis. Diffuse cone dysfunction is diagnostic of cone dystrophy. Diseases such as sector retinitis pigmentosa, which destroys only foc al regions of retina, may reduce bwave amplitude, but the shape and timing of the waveforms (being generated by th e remaining healthy areas of retina) is usually normal. A8 A loss of red-sensitive pigment results in a red-green color confusion defect and also makes the longer wavelength portion of the spectrum appear darker than normal. Because cone photo receptors are not actually missing, acuity and photosensitivity are normal. A9 A subnormal EOG in the setting of a normal ERG is a consistent, classic findi ng in Best disease. It can also be seen occasionally with the various forms of adult-onset pattern dystrophies. In retin itis pigmentosa, both the ERG and EOG are subnormal. In rubella retinopathy, the RPE is diffusely affected but the EOG is normal. A10 The vast majority of cases of Stargardt disease are autosomal recessive. Bes t vitelliform dystrophy, familial drusen, pattern macular dystrophies, Sorsby macular dystrophy are typically inhe rited in autosomal dominant fashion. A11 A constant diagnostic feature of congenital X-linked retinoschisis is foveal schisis. There is 100% penetrance for foveal schisis in this disorder, even in young children. The a-wave is typic ally normal whereas the b-wave is reduced, reflecting the Muller cell dysfunction thought to play a role in pathog enesis. Fluorescein leakage is absent in foveal schisis. Peripheral retinoschisis and pigmentary changes are each pres ent in approximately half of affected patients. A12 Monitoring for hydroxychloroquine retinopathy. a. The earliest signs of toxicity include relative paracentral scotomata and sub tle paracentral macular pigment granularity. b. The "safe" daily dose is considered to be less than 6.5 mg/kg/day (based on l ean body weight). c. Baseline evaluation should include complete ophthalmologic examination, fundu s photographs, and evaluation of the central visual field with a red test object. d. In addition to higher daily and cumulative doses, other risk factors for reti nal toxicity include obesity, kidney or liver disease, older age, and possibly concomitant retinal disease. e. Although the incidence of retinal toxicity from hydroxychloroquine is very lo w in the United States, it is of serious concern because associated visual loss rarely recovers and may even prog

ress after the drug is discontinued. Annual screening examinations are recommended for everyone in high er risk categories (including all patients with more than 5 years of usage). A13 Pigmentation in retinitis pigmentosa (RP) is variable, and many patients hav e few or no bone spicules. Rhodopsin gene abnormalities account for only about 30% of dominant RP, and most recessive RP has not been genetically defined. From a clinical standpoint, the ERG is the most critical me asure because it documents the diffuse photoreceptor damage that defines the group of hereditary dystrophies th at we call RP. Most RP patients have mild tritan (blue-yellow) color deficiency. Small tubular fields are a char acteristic late finding in RP, but they are not pathognomonic, and many younger patients still have large areas of periphera l vision. A14 Fifty percent of rhegmatogenous retinal detachments associated with blunt tr auma in young eyes is found within 8 months. Young eyes rarely develop an acute rhegmatogenous retinal detac hment following blunt trauma because their vitreous has not yet undergone syneresis. Therefore, the vitreous provides an internal tamponade. Over several months, however, the vitreous over a tear may liquefy, permitting f luid to pass through the break to detach the retina. A15 The Joint Statement recommends that infants meeting any of these criteria un dergo at least 2 screening examinations for retinopathy of prematurity. a. A birth weight less than 1500 grams. b. A gestational age of 28 weeks or less. c. A birth weight between 1500 and 2000 grams and an unstable clinical course. A16 Scatter laser treatment is indicated in patients with high-risk PDR. If a ca taract is present, the ideal timing for laser application is 1-2 months pre-cataract extraction to allow the proliferati ve changes time to respond. A17 PIC PICis a bilateral condition that typically affects young, otherwise healthy, wom en who have a mild to moderate degree of myopia. Choroidal neovascularization remains a major cause of visual l oss in affected individuals. A18 DUSN, although rare, is an important disease to consider, as it is a treatab le cause of severe visual loss that often affects children. If left untreated, it will lead to widespread RPE disrup tion and is frequently mistaken for "unilateral retinitis pigmentosa." The condition has been described in almost ev ery region of the world and is not associated with any specific travel history.

A19 Pneumatic retinopexy Pneumatic retinopexyworks by tamponade of causative breaks and not on buoyant fo rces on the retina itself. Chronic subretinal fluid typically has delayed resorption, and pneumatic procedures have a poorer success rate in this setting. Chronic detachments are a relative contraindication for pn eumatic retinopexy. A20 Typically, retinal veins are dilated with both CRVO and carotid artery occlu sive disease, but often they are tortuous only in CRVO. Ophthalmodynamometry measures the retinal artery pressure , which is normal in CRVO and low in carotid artery occlusive disease. A21 is characterized by, enlargement of the physiologic Multiple evanescent whit e dot syndrome (MEWDS) Multiple evanescent white dot syndrome (MEWDS) blind spot on visual field testing, typically presents with unilateral photopsia s and loss of vision in young females with myopia, absence of cell in the anterior chamber, granular appearance of the fove a. The hyperfluorescent spots in MEWDS are actually wreathlike clusters of smaller hyperfluorescent dots and not individual spots arranged in a wreathlike configuration around the fovea. A22 In a randomized, controlled clinical trial, pneumatic retinopexy provided sl ightly better visual outcome than scleral buckle in patients with macula-involving rhegmatogenous retinal detachme nts of less than 14-day duration. Visual acuity outcome was slightly superior in patients with macula-involving rh egmatogenous retinal detachments of less than 14-day duration who underwent pneumatic retinopexy than in those patie nts who underwent scleral buckling primarily. Only patients with a causative break(s) in the superior two thirds of the retina were included in the study. Anatomic success rates were slightly greater in patients undergoing prima ry scleral buckle, but visual outcome was not affected in patients who underwent unsuccessful pneumatic retinopexy and subsequently underwent scleral buckle procedure. A23 No evidence exists that APMPPE responds to systemic steroid therapy. APMPPE, although typically bilateral, may occur in one eye or be highly asymmetric. Typically a monophasic disease, a recurrent or relentless course may occur and has sometimes been termed "ampiginous choroidopathy." A24 Retinal dialysis is usually treated in phakic patients even when asymptomati c. Atrophic holes and operculated tears are treated only in special circumstances. A25 The visual prognosis is generally good after treatment with pars plana vitre ctomy and removal of the retained lens fragments. In eyes with medium to large amounts of retained lens fragments, marked intraocular inflammation is common; secondary glaucoma is also relatively common. Retinal detachment is less common but has been reported in approximately 15% of these eyes in large published series. A26 Tobacco dust, also known as Shafer's sign, is manifested by small clumps of pigmented cells in the vitreous and is practically diagnostic of rhegmatogenous retinal detachment. Tractional r etinal detachments nearly always have a concave surface that is smooth, rather than corrugated; they almost never extend to the ora serrata. Sickle cell retinopathy is a well-known cause of tractional retinal detachment.

A27 In treating CNV associated with ocular histoplasmosis, the ophthalmologist c an decrease the risk of recurrent CNV by attaining a uniform white intensity of the area of photocoagulation at le ast as great as the minimal intensity standard published by the Macular Photocoagulation Study (MPS) and covering the entire lesion with laser treatment. Only one prospective trial compared one laser wavelength to another w hen treating CNV: the MPS subfoveal trials of CNV associated with AMD. In these trials, wavelength was not shown to affect the incidence of recurrence. Although the duration of treatment should be relatively long to crea te an intense lesion without suddenly breaking through Bruch's membrane, duration has not been shown to affect the rat e of recurrence. However, failure to cover the entire lesion or to achieve a white intensity at least as great as the minimal intensity standards published by the MPS are each factors that independently increased the likelihood of devel oping persistent CNV. A28 Laser treatment of subfoveal lesions should be undertaken only in the presen ce of some classic CNV. However, occult CNV can also be present as long as there is evidence of classic CNV. Because the laser treatment should cover the entire lesion, the boundary of the entire lesion should be well demarcated so that the treating ophthalmologist can clearly recognize the extent of the CNV. Blood can be presen t as a lesion component provided that the area of CNV is greater than any areas of blood or blocked fluorescence (not from blood) or serous detachment of the RPE. A29 Diffuse hemangiomas are associated with Sturge-Weber syndrome, whereas circu mscribed ones are not. Both types are associated with serous detachments of the retina. A30 An area of neovascularization in a patient with age-related macular degenera tion shows leakage during fluorescein angiography, additional fluorescein characteristic is needed to help define the neovascularization as "classic" is the vessels are seen early in the angiogram. The additional piece o f information needed is the ability to see the vessels early in the fluorescein angiogram. Being "well defined" is n ot a necessary characteristic. Soft drusen occur in age-related macular degeneration but are not a requisite fi nding for the designation "classic." A31 The maculopathy caused by Tamoxifen is characterized by crystalline deposits and sometimes macular edema. Moderate degrees of both functional loss and anatomic degenerative change s can occur. A crystalline maculopathy may also be seen after ingestion of high doses of canthaxanthin, a c arotenoid available in health food stores used to simulate tanning. Canthaxanthine retinopathy is generally asympto matic, and the deposits resolve when the drug is stopped. I. . REFRACTIVE SURGERY REFRACTIVE SURGERY

A1 Topographic changes in keratoconus include central and inferior corneal steep ening, hemimeridional asymmetry and a high inferior-superior (I-S) number. LASIK should be avoided in keratoconu s to prevent progressive ectasia of the cornea. A2 Optical aberrations consisting of sphere and cylinder are considered lower or der (second-order) aberrations. Other optical aberrations are called higher-order aberrations. A3 In conventional non-wavefront-guided laser treatment of myopia, the central c ornea is flattened. Defocus (a secondorder aberration) is corrected at the expense of induced spherical (high-order) aberrations. A4 The following is true after conventional laser ablations for myopia (LASIK, L ASEK, or PRK). a. Lower-order (second-order) aberrations are generally reduced. b. Higher-order aberrations often increase. c. The central corneal curvature is flattened and the corneal apical radius of c urvature is increased. d. Corneal asphericity changes often accompany myopic laser ablations. A5 For the same amount of intended dioptric correction, increasing the size of t he ablation zone results in deeper treatment. Munnerlyn's approximation of the ablation depth per diopter of correc tion = diameter²/ 3, where the diameter is in mm and the depth is in .m. Thus the ablation depth is proportiona l to the square of the diameter. The depth of tissue ablation per diopter of correction is greater for wider abla tion zones. A6 For low amounts of correction (
View more...

Comments

Copyright ©2017 KUPDF Inc.
SUPPORT KUPDF